2013 ITE

Lakukan tugas rumah & ujian kamu dengan baik sekarang menggunakan Quizwiz!

70. Which one of the following is the recommended duration of thromboprophylaxis following total hip arthroplasty, starting from the day of surgery and including outpatient prophylaxis? A) 7 days B) 14 days C) 35 days D) 60 days E) 90 days

ANSWER: C For patients undergoing major orthopedic surgery, the American College of Chest Physicians recommends outpatient thromboprophylaxis for a duration of up to 35 days. Older recommendations for 10-14 days of prophylaxis were based on studies performed when this was the usual hospital stay. This is still recommended as the minimum length for prophylaxis, but a longer period of time is preferred.

22. A 50-year-old white male is injured while mountain biking. The CT scan of the patient's abdomen shown below is most consistent with which one of the following? A) Rupture of the spleen B) Subcapsular hematoma of the spleen C) Subcapsular hematoma of the kidney D) Ruptured hollow viscus E) Herniated vertebral disc

ANSWER: C The CT scan shows a subcapsular hematoma of the kidney. This is considered a grade I injury and does not require surgical treatment.

15. Which one of the following is most consistent with a diagnosis of asthma? A) Reduced FEV1 and a decreased FEV1/FVC ratio B) Reduced FEV1 and a normal FEV1/FVC ratio C) Reduced FEV1 and an increased FEV1/FVC ratio D) Reduced FVC and a normal FEV1/FVC ratio E) Reduced FVC and an increased FEV1/FVC ratio

ANSWER: A Asthma is typically associated with an obstructive impairment that is reversible with short-acting bronchodilators. A reduced FEV1 and a decreased FEV1/FVC ratio indicates airflow obstruction. A reduced FVC with a normal or increased FEV1/FVC ratio is consistent with a restrictive pattern of lung function.

75. According to national and international guidelines, which one of the following is the next step for adults with asthma who require therapy with inhaled -agonists more than three times a week? A) Inhaled glucocorticoids B) Inhaled salmeterol (Serevent) C) Sustained-release oral -agonists D) Sustained-release oral theophylline

ANSWER: A Patients who require inhalation therapy with Beta2-adrenergic-receptor agonists more than twice weekly but not daily have mild persistent asthma. Long-term control with inhaled corticosteroids is recommended for adults with persistent asthma.

62. For the prevention of ischemic stroke in patients at low risk for gastrointestinal bleeding, the U.S. Preventive Services Task Force recommends aspirin for A) men age 45-79 B) men age 55-79 C) women age 45-79 D) women age 55-79 E) no one, regardless of sex or age

ANSWER: D The U.S. Preventive Services Task Force has concluded that the net benefit of daily aspirin is substantial in women 55-79 years of age for whom the benefit of ischemic stroke prevention exceeds the harm of an increased risk for gastrointestinal bleeding (SOR A). Aspirin use is recommended in men 45-79 years of age for prevention of myocardial infarction when the potential benefit outweighs the potential harm of gastrointestinal hemorrhage (SOR A).

27. A 58-year-old male has a history of type 2 diabetes mellitus that is not well controlled. He has recently developed mild hypertension that has not been controlled by lifestyle changes. You prescribe lisinopril (Prinivil, Zestril), 20 mg daily, for the hypertension and 2 months later you note that his serum creatinine level has increased from 1.25 mg/dL to 1.5 mg/dL (N 0.64-1.27) and his blood pressure has decreased from 142/88 mm Hg to 128/78 mm Hg. Which one of the following should you do now? A) Continue the current dosage of lisinopril B) Decrease the dosage of lisinopril to 10 mg C) Increase the dosage of lisinopril to 40 mg D) Discontinue lisinopril and initiate chlorthalidone E) Discontinue lisinopril and initiate losartan (Cozaar)

ANSWER: A ACE inhibitors such as lisinopril do not need to be discontinued unless baseline creatinine increases by >30%. (This patient's creatinine increased by 20%.) The current dosage of lisinopril is appropriate, as the blood pressure meets the diabetic goal of <130/80 mm Hg. Small increases in creatinine have been associated with long-term preservation of renal function, and may be a marker of changes in intraglomerular pressure.

90. Which one of the following is best for preventing acute mountain sickness? A) Acetazolamide (Diamox Sequels) started the day before arriving at altitude B) Prednisone started the day before arriving at altitude C) Moderate alcohol consumption on the first day at altitude D) Ascending quickly, then resting to acclimatize before beginning planned activities

ANSWER: A Acute mountain sickness is common in people traveling to altitudes higher than 8200 ft. Symptoms include headache and at least one of the following: nausea or vomiting, anorexia, dizziness or lightheadedness, fatigue or weakness, and difficulty sleeping. Slow ascent is the most effective way to prevent acute mountain sickness. Acetazolamide or dexamethasone can be used for both prevention and treatment. Ataxia and altered mental status are signs of cerebral edema and occur with end-stage acute mountain sickness. This can progress to coma and death and requires prompt treatment and descent. High-altitude pulmonary edema can occur without acute mountain sickness. Alcohol consumption on the first day at altitude can exacerbate acute mountain sickness.

84. A 27-year-old male complains of difficulty sleeping, forgetfulness, numbness, and feeling detached from life. These symptoms started 2 weeks ago after he was in a motor vehicle accident in which his younger brother was killed. He admits to troubling flashbacks to the accident and would like help with these problems. Which one of the following is likely to be most effective for this patient? A) Cognitive-behavioral therapy B) Critical incident stress debriefing C) Supportive counseling D) Fluoxetine (Prozac) E) Propranolol

ANSWER: A Acute stress disorder (ASD) is a condition that develops within 4 weeks of a traumatic experience. The individual with ASD suffers dissociative symptoms including amnesia, a reduction in awareness of his or her surroundings, numbness, and detachment. Patients who suffer from ASD are at higher risk for developing PTSD, but appropriate treatment can reduce this risk. Cognitive-behavioral management (muscle relaxation techniques, gradual introduction to avoided situations, restructuring beliefs about the trauma) aids in the treatment of ASD (SOR A). Cognitive-behavioral therapy has been found to be more effective than supportive counseling. Although pharmacologic management can be helpful, the evidence does not show it to be broadly beneficial. Imipramine, fluoxetine, risperidone, and propranolol have all been used for ASD, but are not recommended for routine use (SOR C). Critical incident stress debriefing is a group activity usually provided shortly after a traumatic event. During the sessions, patients are invited to share their feelings and education is provided on coping with the trauma. There has been no evidence of a positive effect of stress debriefing and it may actually be harmful by interfering with the natural course of recovery (SOR C).

57. A 56-year-old African-American male with long-standing hypertension and a 30-pack-year smoking history has a 2-day history of dyspnea on exertion. A physical examination is unremarkable except for rare crackles at the bases of the lungs. Which one of the following serologic tests would be most helpful for detecting left ventricular dysfunction? A) B-type natriuretic peptide (BNP) B) Troponin T C) C-reactive protein (CRP) D) D-dimer E) Cardiac interleukin-2

ANSWER: A B-type natriuretic peptide (BNP) is a 32-amino acid polypeptide secreted from the cardiac ventricles in response to ventricular volume expansion and pressure overload. The major source of BNP is the cardiac ventricles, and because of the minimal presence of BNP in storage granules, its release is directly proportional to ventricular dysfunction. A BNP test is simple and time efficient, and reliably predicts the presence or absence of left ventricular dysfunction on an echocardiogram.

19. Which one of the following medications is most likely to cause hypokalemia? A) Albuterol (Proventil, Ventolin) B) Doxazosin (Cardura) C) Erythromycin D) Felodipine (Plendil) E) Lisinopril (Prinivil, Zestril)

ANSWER: A Beta-Agonists activate potassium uptake by the cells. This includes bronchodilators and tocolytic agents. Other agents that can induce hypokalemia include pseudoephedrine and insulin. Diuretics, particularly thiazides, can also cause hypokalemia as a result of the renal loss of potassium.

72. You respond to a code blue in the obstetrics department. The patient is a 19-year-old primigravida at 35 weeks gestation, hospitalized with severe preeclampsia. A nurse anesthetist has placed an oral airway and is administering 100% oxygen to the apneic patient. She reports no difficulty ventilating the patient with a bag and valve, and no gagging with oral airway insertion. The patient's blood pressure is 100/60 mm Hg and her pulse rate is 70 beats/min and regular. Her pupils are equal and sluggishly reactive, and she is flaccid and areflexic. The patient had been treated with a magnesium sulfate infusion and a recent bolus of labetalol. Which one of the following medications should you administer initially? A) Calcium gluconate B) Fosphenytoin C) Labetalol D) Lorazepam (Ativan) E) Dopamine

ANSWER: A During the treatment of severe preeclampsia with intravenous magnesium, the occurrence of apnea and areflexia is most consistent with magnesium toxicity. In addition to hemodynamic support, calcium infusion is recommended as an antidote. Calcium chloride can be used if a central line has been established. Calcium gluconate would be safer with a peripheral intravenous site. Lorazepam, phenytoin, and fosphenytoin are less useful in preventing eclamptic seizures than magnesium. Labetalol is not indicated given the patient's current blood pressure level. Dopamine, a pressor agent, is not indicated in this scenario, and could aggravate the patient's preeclampsia.

11. Which one of the following tumors is most likely to cause hormonally induced hypercalcemia? A) Squamous cell carcinoma of the lung B) Pheochromocytoma C) Medullary thyroid cancer D) Prostatic carcinoma

ANSWER: A Hypercalcemia due to malignancy has a poor prognosis. Up to 80% of cases are due to secretion of parathyroid hormone-related protein. This is most common with squamous cell carcinomas. Breast cancer, lymphomas, and multiple myeloma may cause hypercalcemia as a result of osteolytic activity at the site of the metastasis. Small cell carcinoma of the lung is a major cause of the syndrome of inappropriate secretion of antidiuretic hormone (SIADH) and may also cause Cushing syndrome. Prostate cancer can also cause SIADH, and thyroid cancer can cause Cushing syndrome.

56. A 76-year-old white male with heart failure is admitted to the hospital for the third time in a year. He responds to treatment with intravenous furosemide (Lasix), oxygen, and morphine. When he is discharged, his medications include carvedilol (Coreg), 25 mg twice daily; furosemide, 40 mg daily; and lisinopril (Prinivil, Zestril), 40 mg daily. He is also placed on a low-salt diet. Which one of the following is most likely to help prevent future admissions and decrease overall medical costs for this patient during the next year? A) Case management by a heart failure specialist nurse B) Nursing home admission C) Adding amiodarone (Cordarone) D) Increasing the dosage of lisinopril

ANSWER: A Many elderly patients with heart failure are hospitalized repeatedly at short intervals. As described in a 2012 Cochrane review of 25 randomized, controlled trials, there is now good evidence that case management interventions led by a heart failure specialist nurse reduce heart failure readmissions, all-cause readmissions, and all-cause mortality. Case management interventions include home care, telephone calls, patient education, self-management, and face-to-face visits. It is not possible to say which specific interventions were optimal, but telephone follow-up was a common component in most of these trials. Multidisciplinary interventions may also be effective. For patients treated with lisinopril, 40 mg/day is the maximum amount recommended for heart failure. While Beta-adrenergic blockers and aldosterone antagonists are used in the treatment of heart failure, prophylactic amiodarone would not be expected to help and may be harmful. Admission to a nursing home may reduce hospitalization, but it would also increase overall costs of care.

92. Which one of the following cardiovascular changes is a recognized age-related effect? A) Decreased maximal heart rate with exercise B) Decreased myocardial collagen C) Decreased myocardial mass D) Increased left ventricular compliance E) Increased heart rate at rest

ANSWER: A Maximal heart rate with exercise generally decreases with age. A frequently used formula for predicting maximal heart rate is 220 minus age, with a correction factor of 0.85 often applied for females, who have a lower peak heart rate and a more gradual decline. Myocardial collagen and mass both increase with age. The increase in collagen may play a role in decreasing left ventricular compliance. The resting heart rate, like the maximal exercising heart rate, decreases with normal aging. Tachycardia at rest may suggest a pathologic state.

29. A 22-year-old primigravida presents for routine prenatal care at 18 weeks gestation. She is frustrated because of increased pigmentation on her face consistent with melasma (chloasma). Which one of the following would you recommend for this patient? A) Use of a high-potency broad-spectrum sunscreen B) Use of hydroquinone for 4 weeks C) Postpartum use of oral contraceptives D) Avoiding future use of topical retinoids E) Increased surveillance for skin cancer beginning at age 40

ANSWER: A Melasma is a very common condition in pregnancy and is due to hyperpigmentation related to normal hormonal changes that accompany pregnancy. It can also be caused by oral contraceptives and is more common in dark-skinned persons. High-potency broad-spectrum sunscreens may help prevent melasma, or at least prevent worsening of the condition (SOR C). Topical retinoids, hydroquinone, and corticosteroids can also be helpful, but are usually reserved for postpartum use and require months of treatment. Other treatments include azelaic acid, chemical peels, kojic acid, cryosurgery, and laser treatment (SOR B). Melasma usually improves spontaneously after delivery, but it may be prolonged or worsened by oral contraceptive use. It does not increase the risk of developing skin malignancies.

77. A 7-year-old male presents with a 3-day history of sore throat, hoarseness, fever to 100°F (38°C), and cough. Your examination reveals injection of his tonsils, no exudates, shotty lymphadenopathy, and normal breath sounds. Which one of the following would be most appropriate? A) Symptomatic treatment only B) Empiric treatment for streptococcal pharyngitis C) A rapid antigen test for streptococcal pharyngitis D) A throat culture for streptococcal pharyngitis E) An office test for mononucleosis

ANSWER: A Pharyngitis is a common complaint, and usually has a viral cause. The key factors in diagnosing streptococcal pharyngitis are a fever over 100.4°F, tonsillar exudates, anterior cervical lymphadenopathy, and absence of cough. The scenario described is consistent with a viral infection, with no risk factors to make streptococcal infection likely; therefore, this patient should be offered symptomatic treatment only. Testing for other infections is not indicated unless the patient worsens or does not improve.

83. A 58-year-old healthy white female sees you for a routine visit. She is monogamous with her husband, is a nonsmoker, has two alcoholic drinks a week, and has mild GERD. Her BMI is normal. She takes an over-the-counter H2-blocker and a multivitamin with calcium. She had a normal mammogram 1 month ago and a negative colonoscopy at age 53. She has never had a DXA scan or screening for ovarian cancer. Her family history is noncontributory. According to the U.S. Preventive Services Task Force, you should recommend A) HIV screening B) CA-125 testing for ovarian cancer screening C) DXA for osteoporosis screening D) colonoscopy for colorectal cancer screening

ANSWER: A The U.S. Preventive Services Task Force (USPSTF) and the Centers for Disease Control and Prevention (CDC) recommend that all adults age 65 years and under be screened for HIV regardless of risk factors. The USPSTF does not recommend routine screening for ovarian cancer with a bimanual examination, transvaginal ultrasonography, or CA-125 testing. The USPSTF recommends that women age 65 and older be screened for osteoporosis with a DXA scan. Women younger than age 65 should be screened only if their risk of fracture is equal to or greater than a 65-year-old white female with no additional risk factors. The USPSTF recommends that women age 50-75 be screened for colorectal cancer with colonoscopy every 10 years OR with flexible sigmoidoscopy every 5 years plus fecal occult blood testing (FOBT) every 3 years OR with FOBT annually.

43. Which one of the following conditions presents an unacceptable health risk for combined oral contraceptive use? A) Migraine with aura B) Endometrial hyperplasia C) Breastfeeding 1-6 months post partum D) Chronic hepatitis C E) Previous laparoscopic banding weight-loss surgery

ANSWER: A The World Health Organization (WHO) publishes the medical eligibility criteria for contraceptive use as a guideline for the appropriate use of contraceptives. There are four categories that define the appropriateness of contraceptive use in women with certain medical problems: Category 1: A condition for which there is no restriction for the use of the contraceptive method. Category 2: A condition for which the advantages of using the method generally outweigh the theoretical or proven risks. Category 3: A condition for which the theoretical risk or proven risks usually outweigh the advantages of using the method. Category 4: A condition that represents an unacceptable health risk if the contraceptive method is used. A history of migraine with aura is classified as category 4 for oral contraceptives. Women with a history of migraines are 2-4 times as likely to have a stroke compared to women without migraines, and women who have an aura associated with their migraines are at even higher risk. Migraine without aura is classified as category 2 in women younger than 35 and category 3 in women 35 or older. Nonmigrainous headaches are category 1, as is chronic hepatitis C. Combined oral contraceptive use does not appear to increase the rate or severity of cirrhotic fibrosis and there is no increased risk for hepatocellular carcinoma. Combined oral contraceptives are not recommended for use in women with acute hepatitis C. Breastfeeding is considered category 2 by the CDC and category 3 by WHO. There is conflicting evidence about the effects on the volume of breast milk in women who are on combined oral contraceptives, but the concerns are mainly during the first month of the postpartum period. There have not been any demonstrated adverse health effects in infants exposed to combined oral contraceptives through breast milk. Laparoscopic banding weight-loss surgery is category 1 for combined oral contraceptive use. Evidence shows no significant decrease in the effectiveness of oral contraceptives in women who have had this surgery. Combined oral contraceptive use in patients with endometrial hyperplasia is category 1. Combined oral contraceptives have been used to decrease the risk for endometrial cancer.

94. A 13-year-old female is brought to your office for evaluation of school difficulties and depressed mood. Her mother and older sister have both been diagnosed with depression. After a thorough history and physical examination, you diagnose major depressive disorder. You arrange for the patient to receive cognitive-behavioral therapy, but after 6 weeks her condition is only minimally improved. Which one of the following medications would be appropriate to add to this patient's treatment plan at this point? A) Fluoxetine (Prozac) B) Imipramine (Tofranil) C) Lithium D) Venlafaxine

ANSWER: A The diagnostic criteria for depression are the same for children and adults, although the manner in which these symptoms present may be different. Adolescents with depression are more likely to experience anhedonia, boredom, hopelessness, hypersomnia, weight change, alcohol or drug use, and suicide attempts. Psychotherapy should always be included as part of a treatment plan for depression in adolescents. Cognitive-behavioral therapy and interpersonal therapy are two modalities that have been proven effective in the treatment of adolescent depression. Medications should be considered for more severe depression or depression failing to respond to psychotherapy. A Cochrane review found that fluoxetine was the only agent with consistent evidence of effectiveness in decreasing depressive symptoms in adolescents. Consensus guidelines recommend fluoxetine, citalopram, or sertraline as first-line treatments for moderate to severe depression in children and adolescents. Escitalopram is also licensed for the treatment of depression in adolescents age 12 or older. All antidepressants have a boxed warning regarding an increased risk of suicide; therefore, close monitoring is recommended to assess for suicidality and other adverse effects, such as gastrointestinal effects, nervousness, headache, and restlessness. Tricyclic antidepressants were previously used to treat depression in children, but studies have shown little to no benefit in adolescents and children.

39. Which one of the following vaccines is CONTRAINDICATED in immunocompromised adults? A) Herpes zoster B) Human papillomavirus C) Meningococcal D) Pneumococcal polysaccharide E) Tdap

ANSWER: A The herpes zoster vaccine is the only live-attenuated virus vaccine listed, and is therefore the one contraindicated in immunodeficient patients. Human papillomavirus, Tdap, meningococcal, and pneumococcal polysaccharide vaccines are not live or live-attenuated vaccines, and may be given to immunocompromised patients.

88. An 85-year-old navy veteran presents to your office with a complaint of cough and dyspnea with exertion. He spent his entire career in ship maintenance and repair, and retired from the navy at the age of 45. His chest radiograph shows pleural plaques. He has a 20-pack-year smoking history, but quit at the age of 39. You suspect his problem is due to occupational exposure to which one of the following? A) Asbestos B) Beryllium C) Iron oxide D) Silica E) Uranium

ANSWER: A The inhalation of asbestos fibers may lead to a number of respiratory diseases, including lung cancer, asbestosis, pleural plaques, benign pleural effusion, and malignant mesothelioma. High-risk populations for asbestos exposure include individuals who worked in construction trades or as boilermakers, shipyard workers, or railroad workers, as well as U.S. Navy veterans. The occupational history helps to guide clinical suspicion in these high-risk populations. This patient is a retired U.S. Navy veteran who spent his entire career in ship maintenance and repair. The patient history is not consistent with berylliosis, silicosis, or uranium exposure. Berylliosis is an occupational disease related to mining and manufacturing. Silicosis is seen in sandblasters, miners, persons who have worked with abrasives, and several other occupations. Uranium exposure occurs after nuclear reactor leaks or blasts. Uranium compounds are also used in photography and as dyes or fixatures. The chemical toxicity involves nonmalignant damage to alveolar cells. Iron oxide exposure is not known to be related to lung disease.

35. An 85-year-old male is admitted to a nursing home due to weakness, debility, and limitation of activities of daily living (ADLs) after being hospitalized for acute community-acquired pneumonia. He previously lived with his wife independently and his goal is to return home when he is strong enough. He has a history of coronary artery disease, type 2 diabetes mellitus controlled with diet, hypertension, and chronic diastolic heart failure, but he has no symptoms related to these chronic problems. His appetite is poor and he has lost a significant amount of weight. His admission diet order from the hospital was a cardiac diet. Which one of the following would be the most appropriate diet for this patient? A) A regular diet B) An American Heart Association diet C) A diet with no added salt D) An 1800-calorie/day American Diabetes Association diet E) A diet with no concentrated sweets

ANSWER: A This patient should be provided with a regular diet, which may promote weight gain in nursing-home residents with unintentional weight loss. Malnutrition and unintentional weight loss are significant problems in nursing-home residents and lead to multiple complications, including pressure ulcers and infections. The American Dietetic Association recommends liberalizing diets to improve nutritional status and quality of life in older adults. A small study demonstrated equivalent glycemic control in nursing-home residents who ate a regular diet compared to those who ate a restricted American Diabetes Association diet (SOR C). Low-salt and low-cholesterol diets are unpalatable and are often associated with protein-energy malnutrition and postural hypotension in older persons. Special diets should be avoided whenever possible in nursing-home patients.

25. An otherwise healthy 1-year-old male is brought to your office because of increased respiratory effort, wheezing, and rhinorrhea. He has no fever. On examination he is found to have an increased respiratory rate and mild retractions. A chest film shows no foreign body or infiltrates. His O2 saturation is 94%. Management should include which one of the following? A) A trial of nebulized albuterol (AccuNeb) B) Nebulized epinephrine (Asthmanefrin) C) Oxygen D) Antibiotics E) Corticosteroids

ANSWER: A This presentation is consistent with bronchiolitis, which is a response to a viral respiratory infection. American Academy of Pediatrics guidelines for the management of bronchiolitis do not recommend routine use of any treatment, recommending instead that the choice be based on the specific needs of the child. If the child responds to a trial of albuterol, then treatment can be continued; otherwise, evidence shows no benefit. Antibiotics are indicated for signs of bacterial infection. is <90%. Corticosteroids have not been shown to be of benefit. Oxygen is indicated if the O2 saturation

2. A 36-year-old male with a history of ankylosing spondylitis and atrial fibrillation presents with a 3-week history of cough with hemoptysis, anorexia, night sweats, and an 11-lb weight loss. On examination he has rales in the right upper lobe, but there is no lymphadenopathy or hepatosplenomegaly. A chest radiograph shows a cavitary lesion in the right lung apex, with mediastinal hilar lymphadenopathy. His chronic disease symptoms have been well controlled with a combination of meloxicam (Mobic), adalimumab (Humira), esomeprazole (Nexium), ondansetron (Zofran), docusate sodium (Colace), and amiodarone (Cordarone). Which one of the patient's medications is most likely contributing to his current problem? A) Adalimumab B) Amiodarone C) Esomeprazole D) Meloxicam E) Ondansetron

ANSWER: A Tumor necrosis factor (TNF) inhibitors are currently approved by the U.S. Food and Drug Administration (FDA) for the treatment of rheumatic diseases such as rheumatoid arthritis, ankylosing spondylitis, psoriatic arthritis, and juvenile idiopathic arthritis. All drugs in this class carry an FDA black-box warning about the potential for developing primary tuberculosis or reactivating latent tuberculosis. These drugs are also associated with an increased risk for invasive fungal infections and opportunistic bacterial and viral diseases. The FDA also warns of reports of lymphomas and other malignancies in children and adolescents taking these drugs. A PPD skin test should be performed prior to initiating PNF-inhibitor therapy. An induration of 5 mm or greater with tuberculin skin testing should be considered a positive test result when assessing whether treatment for latent tuberculosis is necessary prior to PNF-inhibitor use, even for patients previously vaccinated with bacille Calmette-Guérin (BCG) (SOR B). Ankylosing spondylitis patients may develop fibrosis of the upper lung fields with long-standing disease, but esomeprazole, ondansetron, and meloxicam do not cause reactivation of tuberculosis. Amiodarone is associated with a subacute cough and progressive dyspnea due to pulmonary toxicity (patchy interstitial infiltrates).

95. Which one of the following medications used in the treatment of osteoporosis can also be used to treat the pain associated with acute and chronic vertebral compression fractures? A) Calcitonin-salmon (Miacalcin) B) Raloxifene (Evista) C) Risedronate (Actonel) D) Teriparatide (Forteo) E) Zoledronic acid (Reclast)

ANSWER: A While all of the medications listed can be used to treat osteoporosis, only calcitonin-salmon is useful in the management of pain associated with acute or chronic vertebral fractures. Calcitonin is an antiresorptive agent that has been shown to decrease the risk of vertebral fractures, but it is not considered a first-line treatment for osteoporosis because there are more effective agents. However, it does have modest analgesic properties that make it useful in the treatment of the pain associated with vertebral fractures.

68. You see a 55-year-old female for the first time. She has a 2-year history of chronic daily cough; thick, malodorous sputum; and occasional hemoptysis. She has been treated with antibiotics for recurrent respiratory infections, but is frustrated with her continued symptoms. She has never smoked. Her FEV1/FVC ratio is 60% and CT shows bronchial wall thickening and luminal dilation. The most likely diagnosis is A) emphysema B) bronchiectasis C) chronic bronchitis D) bronchiolitis E) asthma

ANSWER: B Bronchiectasis is an illness of the bronchi and bronchioles involving obstructive and infectious processes that injure airways and cause luminal dilation. In addition to daily viscid, often purulent sputum production with occasional hemoptysis, wheezing and dyspnea occur in 75% of patients. Emphysema and chronic bronchitis, forms of COPD, also cause a decreased FEV1/FVC ratio, but the sputum is generally mucoid and luminal dilation of bronchi is not characteristically present. Bronchiolitis is usually secondary to respiratory syncytial virus infection in young children. Asthma is not characterized by the sputum and CT findings seen in this patient.

80. A 70-year-old male with long-standing poorly controlled hypertension presents with a 6-month history of fatigue and dyspnea on exertion. He lives in a rural area and continues to be active outdoors but has less stamina. He denies edema, orthopnea, chest pain, and palpitations. His only medication is hydrochlorothiazide. On examination the patient's pulse rate is 80 beats/min, his blood pressure is 160/90 mm Hg, and his O2 saturation is 97% on room air. You hear a grade 2/6 midsystolic ejection murmur, which is loudest at the right upper sternal border but is also heard at the lower left sternal border. An S4 is heard at the apex, with the point of maximal intensity at the anterior axillary line of the 5th intercostal space. The S2 is not split but is a single sound. There is no S3. His lungs are clear and there is no peripheral edema. Which one of the following conditions is the most likely cause of the auscultatory findings? A) Aortic regurgitation B) Aortic stenosis C) Mitral regurgitation D) Mitral stenosis E) Mitral valve prolapse

ANSWER: B This patient has aortic stenosis, which is most likely of the degenerative, age-related type. An echocardiogram should be included in this patient's evaluation, but the diagnosis can still be made on the basis of the auscultatory findings. The second heart sound (closure of the aortic valve then the pulmonic valve) is normally split, consisting of two distinct components during inspiration. As the aortic valve becomes more rigid with the progression of aortic stenosis, the valve closures occur simultaneously or even become paradoxically split during expiration. The murmur of aortic stenosis is a midsystolic ejection-type murmur, heard loudest at the right second intercostal space parasternally. It may radiate to the right carotid, and to a lesser degree may also be heard in the fourth intercostal space at the left lower sternal border. An S3 gallop is heard only late in the progression of aortic stenosis and is associated with left ventricular dilatation and heart failure. The murmur of aortic regurgitation is a diastolic descrescendo blowing-type murmur best heard at the left fourth intercostal space at the lower left sternal border, with the patient sitting and leaning forward. Mitral valve murmurs are typically located at the heart apex or even in the left infra-axillary area. Mitra l valve prolapse is characterized by a prominent systolic click, often with a brief mitral regurgitation murmur.

4. Which one of the following accurately describes the classic rash of erythema migrans? A) Scattered individual purple macules on the ankles and wrists B) An annular rash with a bright red outer border and partial central clearing C) A dry, scaling, dark red rash in the groin, with an active border and central clearing D) A diffuse eruption with clear vesicles surrounded by reddish macules E) A migratory pruritic, erythematous, papular eruption

ANSWER: B An annular rash with a bright red outer border and partial central clearing is characteristic of erythema migrans. It is important to remember that not all lesions associated with Lyme disease look this way, and that some patients with Lyme disease may not have any skin lesions at all. Rocky Mountain spotted fever causes scattered individual purple macules on the ankles and wrists. A dry, scaling, dark red rash in the groin, with an active border and central clearing, is seen with tinea cruris. A diffuse eruption with clear vesicles surrounded by reddish macules is found in chickenpox. A migratory pruritic, erythematous, papular eruption is most consistent with urticaria.

6. A 27-year-old male requests your advice regarding colon cancer screening. His brother died of colon cancer, which was diagnosed at the age of 40. You suggest that he begin colonoscopy screening A) now B) at age 30 C) at age 40 D) at age 45 E) at age 50

ANSWER: B Average-risk adults should be screened for colon cancer starting at 50 years of age, and high-risk adults either at age 40 or 10 years before the age at which colorectal cancer was diagnosed in the youngest affected relative.

48. An 11-year-old male is brought to your office for evaluation of bilateral posterior heel pain that has occurred for the past few months. He plays basketball and soccer several times a week and the pain begins several minutes into each of these activities. There is no pain at rest or with walking. The patient has not noticed any numbness, tingling, or weakness. On examination you find no swelling or tenderness of the heel or Achilles tendon. Reflexes, strength, and range of motion at the ankle are intact, but he does have bilateral posterior heel pain when you passively dorsiflex the ankles. Which one of the following is the most likely diagnosis? A) Achilles tendinopathy B) Calcaneal apophysitis C) Plantar fasciitis D) Heel pad syndrome E) Tarsal tunnel syndrome

ANSWER: B Calcaneal apophysitis, also known as Sever disease, is the most common etiology of heel pain in children, usually occurring between 5 and 11 years of age. It is thought that in these children the bones grow faster than the muscles and tendons. A tight Achilles tendon then pulls on its insertion site at the posterior calcaneus with repetitive running or jumping activities, causing microtrauma to the area. There may be swelling and tenderness in this area and passive dorsiflexion may increase the pain. Radiography is usually normal and therefore does not often aid in the diagnosis, but it may reveal a fragmented or sclerotic calcaneal apophysis. Treatment involves decreasing pain-inducing activities, anti-inflammatory or analgesic medication if needed, ice, stretching and strengthening of the gastrocnemius-soleus complex, and the use of orthotic devices. Plantar fasciitis and heel pad syndrome cause pain on the plantar surface of the heel rather than posteriorly. Achilles tendinopathy causes tenderness to palpation of the Achilles tendon. Tarsal tunnel syndrome related to compression of the posterior tibial nerve causes neuropathic pain and numbness in the posteromedial ankle and heel.

51. A 14-year-old male has open and closed comedones without evidence of surrounding inflammation on his face and upper back. Which one of the following is the most appropriate initial treatment? A) Topical antibiotics B) Topical retinoids C) Oral antibiotics D) Oral isotretinoin

ANSWER: B Comedones are noninflammatory acne lesions. Inflammatory lesions include papules, pustules, and nodules. Grading acne based on the type of lesion and severity helps guide therapy. Topical retinoids prevent the formation of comedones and reduce their number, and are indicated as monotherapy for noninflammatory acne. Topical antibiotics are used primarily for the treatment of mild to moderate inflammatory or mixed acne. Oral antibiotics are effective for the treatment of moderate to severe acne. Oral isotretinoin is reserved for treatment of severe, recalcitrant acne.

82. A 64-year-old male comes to your office for evaluation of a persistent rash affecting his groin. It is itchy but not painful and does not affect his daily activities. He has tried over-the-counter antifungal creams without relief. On examination you find well-demarcated, dark red patches in the inguinal region bilaterally. When examined with a Wood's light the area fluoresces coral-red. The most effective treatment for this condition is topical A) ketoconazole (Nizoral) B) erythromycin C) hydrocortisone D) mupirocin (Bactroban) E) terbinafine (Lamisil)

ANSWER: B Coral-red fluorescence on Wood's light examination is typical of infection with Corynebacterium minutissimum, a condition known as erythrasma. This organism commonly complicates intertrigo, often in the groin or interdigital spaces. Erythromycin is the most effective treatment for this bacterial infection.

33. The skin lesion shown below is characteristic of which condition often associated with a drug reaction? A) Id reaction B) Erythema multiforme C) Discoid lupus D) Granuloma annulare E) Pyoderma gangrenosum

ANSWER: B Erythema multiforme (EM) is an acute, often recurrent, inflammatory condition. While it is associated with many different causes such as drugs, infections, and physical agents, it is most commonly associated with acute upper respiratory infections, herpes simplex virus, and Mycoplasma pneumoniae. EM typically occurs in persons 20-40 years of age, with urticarial papules or the classic bull's-eye or target lesions (as shown in this case). The distribution is primarily on the hands (both the dorsi and palms), soles, and extensor aspects of the arms and legs. Lesions may also occur on mucosal surfaces, but in typical EM these are milder than in the more severe Stevens-Johnson syndrome, which is more commonly associated with drug exposure than with infectious agents. The lesions of EM are usually pruritic, but not universally so. They evolve to the typical target forms over 24-48 hr. The central area is often dusky and may be superficially necrotic or vesicular. Partial target lesions may resemble urticaria. The lesions usually heal without scarring over a 10- to 14-day period. An id reaction is a papulovesicular eruption, classically on the sides of fingers, that occurs in response to an intense inflammatory process such as a fungal infection or acute dermatitis in another anatomic area of the body. When that condition resolves, so does the id reaction. Discoid lupus lesions are irregular but roughly round in shape, sharply demarcated, and most commonly found on the face or scalp. They usually develop an adherent thick scale. The duration may be months or years. Granuloma annulare manifests as a ring of firm, flesh-colored or red papules with a more prominent outer ring forming due to central involution. These lesions may last for months to years. The distribution is usually on the dorsolateral aspect of the hands or feet. Pyoderma gangrenosum begins as a tender papule, usually on the lower leg, and evolves to a painful, necrotic, inflammatory lesion that gradually enlarges up to 10 cm. It usually is a manifestation of an underlying systemic inflammatory condition such as inflammatory bowel disease. The lesions last for months to years and heal with scarring.

31. Which one of the following metabolic abnormalities is most likely to be seen in patients with stage 4 kidney disease? A) Hyperaldosteronism B) Hyperparathyroidism C) Hypothyroidism D) Hypogonadism E) Type 2 diabetes mellitus

ANSWER: B Hyperparathyroidism is present in more than half of patients who have a glomerular filtration rate <60 mL/min, and is independently associated with increased mortality and an increased prevalence of cardiovascular disease. In patients with stage 4 chronic kidney disease, current guidelines recommend monitoring of serum calcium and phosphate levels every 3-6 months and bone-specific alkaline phosphatase activity every 6-12 months with the goal of normalizing these values. The other metabolic abnormalities listed are less common than hyperparathyroidism.

21. Which one of the following has been shown to have a beneficial effect for symptoms of the common cold in an adult? A) Diphenhydramine (Benadryl) B) Ipratropium (Atrovent) nasal spray C) Intranasal zinc D) Intranasal corticosteroids E) Systemic corticosteroids

ANSWER: B Ipratropium is the only nasally inhaled anticholinergic recommended by the American College of Chest Physicians for a cough caused by the common cold. One study showed that the nasal formulation decreases rhinorrhea and sneezing, and a Cochrane review found that ipratropium bromide nasal spray improved rhinorrhea but did not help nasal stuffiness (SOR B). Antihistamine monotherapy (either sedating or nonsedating) such as diphenhydramine was no more effective than placebo (SOR A). Corticosteroids have not been found to be effective for the symptoms of a common cold. Intranasal zinc should not be used because it may result in the permanent loss of smell.

64. A 45-year-old male presents with a 3-month history of hoarseness. He denies any other complaints and has not been ill recently. He is not on any medication, has no history of chronic medical problems, and does not smoke cigarettes or drink alcohol. Which one of the following would be the most appropriate management of this patient? A) Voice rest for 1 month B) Laryngoscopy C) A trial of a proton pump inhibitor D) A trial of inhaled corticosteroids E) Oral corticosteroids

ANSWER: B Laryngoscopy should be performed to visualize the larynx and evaluate for vocal cord pathology in a patient whose hoarseness does not resolve within 3 months (SOR C). If a serious condition is suspected for some other reason, laryngoscopy should be performed regardless of the duration of symptoms. If there is a recent history of upper respiratory infection or vocal abuse, then it would be appropriate to recommend voice rest for 2 weeks. Laryngoscopy would then be indicated if the hoarseness did not improve or recurred after voice rest. For patients with symptoms of gastroesophageal reflux, a trial of a proton pump inhibitor is recommended (SOR B). Inhaled corticosteroids, especially fluticasone, may cause hoarseness. Oral corticosteroids do not have a role in the management of hoarseness.

41. A 25-year-old primigravida asks about pain management during labor. You inform her that use of regional analgesia during labor A) increases the likelihood of cesarean delivery B) increases the risk for instrument-assisted vaginal delivery C) provides less pain relief than opioid analgesia D) lowers 1-minute Apgar scores

ANSWER: B Regional analgesia in laboring patients increases the risk of vacuum- or forceps-assisted delivery (relative risk = 1.42; 95% confidence interval, 1.28-1.57; 23 trials; n = 735). Multiple randomized, controlled trials have compared regional analgesia with no analgesia. In a meta-analysis, no statistically significant impact was found on the risk of cesarean delivery, maternal satisfaction with pain relief, long-term backache, or immediate effect on neonatal status as determined by Apgar scores. Regional analgesia provides better pain relief than opioid analgesia.

59. For which one of the following respiratory infections should antibiotic therapy be initiated immediately upon diagnosis? A) Bronchitis B) Epiglottitis C) Laryngitis D) Rhinosinusitis E) Tracheitis

ANSWER: B Many infections of the respiratory tract have a viral etiology, and when this is the case early antibiotic treatment offers little to no benefit. Once the clinical course of a respiratory illness exceeds the expected length for a viral illness, it may be proper to initiate antibiotic treatment for a suspected atypical or secondary bacterial infection. Epiglottitis is one exception to this approach because of the possibility of a bacterial infection, particularly with Haemophilus influenzae type b, that can produce a rapidly worsening, potentially fatal airway compromise. When epiglottitis is suspected based on findings such as hoarseness, dysphagia, stridor, drooling, fever, chills, and respiratory distress, intravenous antibiotic treatment should be instituted immediately, ideally with a Beta-lactam drug that exhibits activity against methicillin-resistant Staphylococcus aureus.

78. Which one of the following is more common in non-Hispanic whites than in the Hispanic population? A) Obesity B) Osteoporosis C) Diabetes mellitus D) Hypertension E) Neurocysticercosis

ANSWER: B Many minority populations in the United States, including Hispanics, suffer health care disparities. Some medical problems are also more common in certain populations. In 2009, Hispanics made up approximately 16% of the U.S. population and were the largest minority. This group is projected to make up 30% of the U.S. population by 2050. The rates of obesity and diabetes mellitus in Hispanics are disproportionately higher than those of non-Hispanic whites. Hypertension is closely linked to obesity, and Hispanics have higher rates of hypertension as well. Neurocysticercosis is the most common cause of seizures in Hispanic immigrants. Risk factors for osteoporosis include female gender, non-Hispanic white ethnicity, smoking, and low BMI.

7. Which one of the following is the major mechanism of action of metformin (Glucophage)? A) Stimulation of pancreatic insulin release B) Inhibition of glucose production by the liver C) Inhibition of carbohydrate absorption in the small intestine D) Improved insulin sensitivity of skeletal muscle

ANSWER: B Metformin has multiple mechanisms of action, but its main effect on serum glucose results from inhibition of gluconeogenesis in the liver. Sulfonylureas and meglitinides stimulate insulin release from the pancreas, and thiazolidinediones sensitize peripheral tissues to insulin. Carbohydrate absorption in the small intestine is inhibited by the "-glucosidase inhibitors.

69. Which one of the following is a classic finding in multiple myeloma? A) Hypokalemia B) Bone pain C) Polycythemia D) Hepatic failure E) Insomnia

ANSWER: B Multiple myeloma can be asymptomatic, but it becomes symptomatic when there is organ damage or other abnormalities, including renal insufficiency, elevated calcium, anemia, and bone disease. The majority of patients have bone pain, but hypocalcemia is not common. Hypokalemia almost never occurs, and both hepatic failure and insomnia are not usual signs of multiple myeloma. Anemia typically occurs either because of renal failure or infiltration of the bone marrow by myeloma cells. Polycythemia does not occur.

66. A previously healthy 74-year-old male presents to the emergency department with a fever and altered mental status. His illness began 2 days ago with symptoms of fever, malaise, body aches, reduced appetite, nausea, and diarrhea. His temperature is 39.6°C (103.3°F) in the emergency department and his examination is nonfocal. Initial laboratory studies include a sodium level of 131 mEq/L (N 135-145) and a WBC count of 14,200/mm3 (N 4500-11,000) with a neutrophilic predominance. Blood and urine cultures are obtained and he is admitted to the hospital for observation. The next morning he develops a productive cough and shortness of breath. You order a chest radiograph, which shows patchy consolidation of the bilateral bases. Which one of the following is the most likely cause of this patient's condition? A) Chlamydophila pneumoniae A) Legionella pneumophila B) Mycoplasma pneumoniae C) Streptococcus pneumoniae

ANSWER: B Pneumonia caused by Legionella pneumophila is commonly preceded by nonspecific systemic symptoms that may lead a clinician to consider other diagnoses. Symptoms may include high-grade fever, malaise, myalgias, anorexia, and headache. Gastrointestinal and neurologic symptoms are also common and include nausea, vomiting, abdominal pain, diarrhea, and confusion. Focal neurologic signs are less common, but have been reported. Localizing respiratory symptoms will typically develop later, most often a dry cough and dyspnea. From this point on the illness resembles a typical pneumonia with fever, productive cough, pleuritic pain, and breathlessness.

40. You are the attending physician at a long-term care facility. A new resident, an 85-year-old female, presents for an initial evaluation. Upon reviewing her history, you find that she is on 18 different medications. Until you can obtain additional history and medical records, you decide to stop or decrease some of her medications and monitor her response. Which one of the following would be most appropriate to stop or decrease initially? A) Sertraline (Zoloft), 25 mg daily B) Acetaminophen/diphenhydramine (Tylenol PM), 500 mg/25 mg daily C) Dipyridamole/aspirin (Aggrenox), 200 mg/25 mg daily D) Digoxin, 0.125 mg every other day E) Omeprazole (Prilosec), 20 mg daily

ANSWER: B Polypharmacy is common in the elderly population, but the use of numerous medications is necessary in some elderly patients. However, some medications have been identified as having a considerably higher potential to cause problems when prescribed to elderly patients. In the case described, acetaminophen/diphenhydramine would be an appropriate medication to stop initially. The older antihistamines cause many adverse CNS effects such as cognitive slowing and delirium in older patients. These effects are more pronounced in elderly patients with some degree of preexisting cognitive impairment. The anticholinergic properties of older antihistamines produce effects such as dry mouth, constipation, blurred vision, and drowsiness. The sedative effect of older antihistamines also increases the risk of falls. Hip fracture and subsequent death have been reported in patients who use older antihistamines such as diphenhydramine. Sertraline is an SSRI, a preferred class for the treatment of depression in the elderly compared to the tricyclic antidepressants, which are associated with several side effects. Dipyridamole is associated with hypotension in elderly patients, but it benefits some individuals by preventing strokes. It can be used in the elderly, but patients should be monitored for side effects. Therefore, until further information is obtained, it is appropriate to continue the dipyridamole/aspirin in this patient. When used in elderly patients with heart failure, digoxin should be given in a dosage no greater than 0.125 mg daily; the low dosage used in this individual should not be considered inappropriate until the reason for its use is clarified. While omeprazole can cause problems in the elderly with long-term use, 20 mg/day is a relatively low dose and the decision to discontinue its use should be delayed until more history is available.

85. A 24-year-old gravida 2 para 1 at 10 weeks gestation presents with fever, myalgias, headache, and malaise. There have been multiple cases of influenza in the community and her influenza swab is positive. Which one of the following is recommended by the Centers for Disease Control and Prevention in this situation? A) Rimantadine (Flumadine) B) Oseltamivir (Tamiflu) C) Acyclovir (Zovirax) D) Supportive therapy only

ANSWER: B The Centers for Disease Control and Prevention (CDC) recommends treatment for persons at higher risk for complications from influenza. Pregnant women and postpartum women within 2 weeks of delivery are considered to be at higher risk. Some data has shown that early antiviral treatment may shorten the duration of symptoms and reduce the risk of complications, especially in these higher risk populations. The CDC recommends against using adamantines due to increased resistance of influenza viruses. Other antivirals such as acyclovir and famciclovir are not active against influenza A and B. The recommended treatment of influenza in pregnancy is one of the neuraminidase inhibitors.

5. A patient with chronic atrial fibrillation treated with dabigatran (Pradaxa) sees you for follow-up. She says she can no longer afford the dabigatran and would like to switch to warfarin (Coumadin). She has normal renal function. Which one of the following would be the most appropriate approach? A) Start warfarin and stop dabigatran when her INR is 2.0-3.0 B) Start warfarin now and stop dabigatran in 3 days C) Stop dabigatran, start warfarin, and start low molecular weight heparin and enoxaparin (Lovenox) every 12 hr until her INR is 2.0-3.0 D) Stop dabigatran for 24 hr and then start warfarin E) Hospitalize the patient, stop dabigatran, start warfarin, and treat with heparin until her INR is 2.0-3.0

ANSWER: B The recommendation for switching to warfarin in a patient treated with dabigatran is to start warfarin 3 days prior to stopping dabigatran. Bridging with a parenteral agent is not necessary. Dabigatran is known to increase the INR, so the INR will not reflect warfarin's effect until dabigatran has been withheld for at least 2 days.

50. A 63-year-old male presents for a preoperative evaluation prior to total knee arthroplasty. He underwent coronary artery angioplasty and stent placement 3 years ago following an episode of angina and an abnormal exercise stress test. His current medications include aspirin, atorvastatin (Lipitor), and lisinopril (Prinivil, Zestril). He runs 2 miles three times/week without difficulty. He has no history of heart failure, diabetes mellitus, kidney disease, or cerebrovascular disease. An EKG and preoperative blood tests, including kidney function tests, are normal. Which one of the following is indicated prior to surgery to decrease this patient's perioperative risk? A) Performing noninvasive cardiac stress testing B) Continuing his statin therapy C) Discontinuing aspirin D) Starting a -blocker E) Starting clopidogrel (Plavix)

ANSWER: B This patient has a Revised Cardiac Risk Index (Goldman Index) score of 1, placing him in a low-risk group for perioperative cardiac complications. Low-risk patients who are able to walk for 2 blocks or climb a flight of stairs without stopping to rest (4 METS) do not need noninvasive cardiac testing. Patients in this risk group who are already on a Beta-blocker should continue it, but adding one preoperatively may increase risk. Stopping aspirin therapy in patients with coronary stents places them at risk for perioperative cardiac events. Surgical bleeding is somewhat increased in patients on aspirin, but differences in the severity of bleeding events and mortality in surgical patients on low-dose aspirin versus controls are minimal. Stopping clopidogrel in patients who have recently undergone coronary stent placement (6 weeks for bare-metal stents, 1 year for drug-eluting stents) markedly increases risk, but there is no need to start clopidogrel in other patients. Perioperative statin therapy should be continued for all patients undergoing surgery. For patients undergoing vascular therapy, statins have been associated with an improvement in postoperative cardiac outcomes.

42. A 28-year-old female visits your office because she is worried about episodic abdominal pain and altered bowel habits that have been present intermittently for the past year. She describes the character of her abdominal pain as "crampy," and says it can become quite severe. Defecation brings some relief, but she sometimes must pass several loose, watery stools before the pain resolves. She occasionally notes mucus in her stool and sometimes feels that evacuation is incomplete even though she is unable to pass more stool. Her body weight is stable, and findings on the abdominal examination are completely normal. Which one of the following diagnostic tests does current evidence support in this case? A) Hydrogen breath testing B) Testing for celiac disease C) Stool testing for ova and parasites D) Thyroid function testing E) CT of the abdomen

ANSWER: B This patient suffers from diarrhea-predominant irritable bowel syndrome (IBS). Her symptoms meet the Rome III criteria for diagnosis with no alarm features. A CBC, serum chemistries, thyroid function studies, stool testing for ova and parasites, and abdominal imaging are all low-yield tests that are not recommended for routine diagnostic evaluation of IBS (SOR C). The association between IBS and bacterial overgrowth in the small intestine is not clear, so routine hydrogen breath testing is not recommended. Routine testing for celiac disease, however, should be considered in patients with diarrhea-predominant or mixed-presentation IBS (SOR C). A systematic review that included more than 4000 patients found that 4% of those with diarrhea-predominant or mixed-presentation IBS had biopsy-proven celiac disease.

9. A 35-year-old nulligravida sees you for preconception counseling. She has hypothyroidism treated with levothyroxine (Synthroid), and her most recent TSH level was in the therapeutic range. She has no symptoms of hypothyroidism. Which one of the following is the patient most likely to require if she becomes pregnant? A) A decreased dosage of levothyroxine B) An increased dosage of levothyroxine C) The addition of liothyronine (Cytomel) D) Substitution of desiccated thyroid hormone preparation (Armour Thyroid) for the levothyroxine

ANSWER: B Thyroid hormone requirements increase during pregnancy. Most women with hypothyroidism who become pregnant require an increased levothyroxine dosage (SOR A). A common recommendation is to have women on fixed daily doses of levothyroxine begin taking nine doses weekly (one extra dose on 2 days of the week) as soon as the pregnancy is confirmed (SOR B). Thyroid function tests should be repeated regularly throughout the pregnancy to guide additional dosage adjustments.

81. A 63-year-old male with a history of alcoholism and compensated hepatic cirrhosis asks if there are pain medications he can use to treat his chronic low back pain and knee and hand osteoarthritis. He also has occasional headaches. He has not used alcohol for several years. Which one of the following medications is CONTRAINDICATED in this patient? A) Acetaminophen B) Gabapentin (Neurontin) C) Naproxen D) Pregabalin (Lyrica) E) Tramadol (Ultram)

ANSWER: C Although patients with chronic mild liver disease may take NSAIDs, they should be avoided in all patients with cirrhosis, due to the risk of precipitating hepatorenal syndrome. Pregabalin and gabapentin are not metabolized by the liver and can be quite helpful. Acetaminophen, while toxic in high doses, can be used safely in dosages of 2-3 g/day. Tramadol is also safe in patients with cirrhosis.

79. A 60-year-old male smoker has lung cancer, and a life expectancy of 4-6 months. His wife is concerned about his state of mind and requests medication for him. His cancer-related pain is generally controlled. When evaluating the patient, which one of the following features would be more characteristic of depression as opposed to a grief reaction? A) Insomnia B) Loss of interest or pleasure in all activities C) Feelings of guilt D) Thoughts of wanting to die E) Psychomotor agitation

ANSWER: B While there is significant overlap in the symptoms of each condition, there are some signs and symptoms that help the family physician determine whether a terminally ill patient is experiencing grief or has major depression. This distinction is important because the terminally ill patient with depression would likely benefit from antidepressant medication, whereas a patient with end-of-life grief is generally best managed without psychotherapeutic medications. The key clinical feature in distinguishing the two conditions is in the pervasiveness of symptoms in depression, particularly the loss of pleasure or interest in all activities. Episodic feelings of guilt, anxiety, and helplessness, and even thoughts of wanting to die can and do occur with grief reactions, but these feelings are not constant and over time the symptoms gradually wane. Terminally ill patients with major depression feel helplessly hopeless all the time, but they often respond to and significantly benefit from antidepressant medication (SOR A).

53. A 57-year-old previously healthy menopausal female presents to your office with a 1-year history of palpitations and an unintentional 10-lb weight loss. A review of systems is negative for tremors or visual changes. Vital signs include a blood pressure of 129/85 mm Hg and a heart rate of 110 beats/min. A physical examination is otherwise unremarkable except for a nontender, diffusely enlarged thyroid with no distinct nodules, and mild proptosis. Laboratory studies are significant for a TSH level<0.01 U/mL (N 0.60-3.30), a free T3 level of 14.51 pg/mL (N 2.0-3.5), and a free T4 level of 4.52 ng/dL (N 0.71-1.40). A thyroid-stimulating immunoglobulin test is positive. In addition to a -blocker, which one of the following is the most appropriate initial management? A) Radioactive iodine ablation B) Thyroidectomy C) Methimazole (Tapazole) D) Propylthiouracil (PTU)

ANSWER: C Methimazole and propylthiouracil (PTU) are the two oral antithyroid medications available. However, because of reports of severe hepatocellular damage, methimazole should be used instead of PTU unless it is contraindicated. Radioactive iodine treatment (131I) is an option, especially for patients who do not achieve remission with antithyroid medications. However, worsening of preexisting orbitopathy is a well-recognized potential complication of 131I treatment, as well as a transient increase in thyroid hormone levels that can precipitate thyroid storm. Thus, patients with elevated free T3 or free T4 levels should be treated with methimazole prior to 131I administration. Thyroidectomy is most often recommended for patients with thyroid nodules and those who are suspected of having cancer or who do not tolerate or refuse alternative forms of therapy. However, antithyroid medication should be given to achieve a euthyroid state prior to surgery in most patients.

100. A 20-year-old male presents with complaints of abdominal pain and diarrhea. He says he often has abdominal cramping that is relieved with defecation. The pain is accompanied by frequent loose, mucous stools, and his symptoms tend to get worse with stress. He says he has tried antidiarrheal medications and antispasmodics, but did not get satisfactory results. Your evaluation leads to a diagnosis of diarrhea-predominant irritable bowel syndrome. Which one of the following would be the most appropriate treatment? A) Fiber supplements B) Neomycin C) Citalopram (Celexa) D) Alosetron (Lotronex) E) Lubiprostone (Amitiza)

ANSWER: C A Cochrane review of 15 studies involving 922 patients showed that antidepressants had a beneficial effect on the symptoms of irritable bowel syndrome (IBS). Both SSRIs and tricyclic antidepressants have shown benefit. Another Cochrane review of 12 randomized, controlled trials did not show any benefit from the use of fiber in any type of IBS. Antibiotics have been shown to have some beneficial effects, but neomycin is used only in constipation-predominant IBS. Lubiprostone is a selective C-2 chloride channel activator and can be used for patients with chronic constipation. Alosetron is a 5-hydroxytryptamine 3 antagonist and is FDA approved to treat severe diarrhea-predominant IBS only in women who have not improved with conventional therapy. Alosetron is associated with uncommon but serious adverse events (ischemic colitis, constipation, death) and its use is restricted in the United States. Other potentially beneficial therapies for IBS include peppermint oil, psychological treatments, exercise, and probiotics.

89. A 15-year-old male is seen in the office for ankle pain. While playing basketball he jumped and landed on the lateral edge of his foot. He had immediate pain and did not continue playing, but was able to walk after the injury. On examination his right ankle has tenderness, swelling, and bruising over the anterior talofibular and calcaneofibular ligaments. There is no bony tenderness. Which one of the following would be most appropriate at this point? A) Taping the ankle for future sports participation B) An elastic compression wrap C) A lace-up ankle support D) A radiograph of the ankle E) A below the knee cast

ANSWER: C A lace-up ankle support reduces pain and recovery time after an ankle sprain (SOR B). The Ottawa Rules state that radiography is required only if there is pain in the malleolar or midfoot zone and either bony tenderness over an area of potential fracture (i.e., distal fibula or tibia, lateral or medial malleolus, base of the fifth metatarsal, or navicular bone) or an inability to bear weight immediately after the injury and when evaluated by a physician. This patient did not have those findings, and therefore would not need a radiograph (SOR A). A cast is not necessary for an ankle sprain. An elastic compression wrap alone is not as effective as a lace-up support. Taping of the ankle for future sports participation can reduce the risk of ankle sprains during sports, but would not be appropriate for an acute injury.

18. A 42-year-old male with a history of chronic hepatitis C develops left leg cellulitis and is treated with cephalexin (Keflex). He returns to your office 5 days later for follow-up, and the cellulitis is responding favorably to treatment. However, the patient has a generalized maculopapular rash and a low-grade fever, which he says began 3 days ago. He also complains of arthralgias. You admit him to the hospital for further evaluation. His serum creatinine level is 3.2 mg/dL (N 0.6-1.5), which is elevated from his baseline level of 0.8 mg/dL. A urinalysis is normal, except for the presence of occasional eosinophils. The remainder of his evaluation, including liver enzyme levels and renal ultrasonography, is normal. Which one of the following is the most appropriate next step in the management of this patient? A) A postvoid residual urine volume B) A hepatitis C viral load and genotype C) Discontinuing cephalexin D) Antibiotics to cover methicillin-resistant Staphylococcus aureus (MRSA) E) Aggressive fluid resuscitation with normal saline

ANSWER: C Acute kidney injury (AKI) is currently defined as either a rise in serum creatinine or a reduction in urine output. Creatinine must increase by at least 0.3 mg/dL, or to 50% above baseline within a 24-48 hour period. A reduction in urine output to 0.5 mL/kg/hr for longer than 6 hours also meets the criteria. Acute interstitial nephritis is an intrinsic renal cause of AKI. These patients are often nonoliguric. A history of recent medication use is key to the diagnosis, as cephalosporins and penicillin analogues are the most common causes. Approximately one-third of patients present with a maculopapular rash, fever, and arthralgias. Eosinophilia and sterile pyuria may also be seen in addition to eosinophiluria. Discontinuation of the offending drug is the cornerstone of management. Although up to 30% of patients with chronic hepatitis C infection have some kidney involvement, acute interstitial nephritis is uncommon. Measuring postvoid residual urine volume is indicated if an obstructive cause for the AKI is suspected. Starting an antibiotic to cover methicillin-resistant Staphylococcus aureus (MRSA) is not indicated.

76. A 30-year-old male presents to your office with a 3-week history of nausea, weight loss, diarrhea, and hematochezia. He states that he has had similar episodes twice in the past and was treated at the local urgent care clinic for infectious diarrhea, with resolution of his symptoms. Your initial laboratory workup is negative for enteric pathogens and you refer the patient for colonoscopy and esophagogastroduodenoscopy with small bowel follow-through. The patient is found to have multiple noncontiguous transmural ulcerations throughout both the small and large intestines. Which one of the following initial management strategies is most likely to induce remission in this patient? A) Laparotomy with colectomy B) Metronidazole (Flagyl) C) Prednisone D) Infliximab (Remicade)

ANSWER: C Inflammatory bowel disease is divided into two categories: Crohn's disease and ulcerative colitis. Noncontiguous or "skip" lesions that are transmural in nature and are found throughout the gastrointestinal tract make a diagnosis of Crohn's disease likely in this patient. Corticosteroids are more effective in inducing remission than placebo and 5-ASA products (SOR A). A Cochrane review revealed no difference between elemental and nonelemental diets with regard to symptom remission (SOR A). Anti-TNF agents such as infliximab should be considered in patients with moderate to severe Crohn's disease who do not respond to initial corticosteroid or immunosuppressive therapy, but these are not recommended for initial treatment. While antibiotics such as metronidazole are widely used for both their anti-inflammatory and anti-infectious properties, controlled trials have not demonstrated their effectiveness. Surgical intervention should be considered in patients with ulcerative colitis, but surgery is not indicated for Crohn's disease.

12. A 32-year-old meat cutter comes to your office with persistent symptoms of nausea, vomiting, and diarrhea, which began about 36 hours ago on the last day of a 5-day Caribbean cruise. His wife was sick during the first 2 days of the cruise with similar symptoms. On the ship they both ate the "usual foods" in addition to oysters. Findings on examination are negative, and a stool specimen is negative for white blood cells. Which one of the following is the most likely cause of his illness? A) Escherichia coli B) Rotavirus C) Norovirus D) Hepatitis A E) Giardia

ANSWER: C Recent reports of epidemics of gastroenteritis on cruise ships are consistent with Norovirus infections due to waterborne or foodborne spread. In the United States these viruses are responsible for about 90% of all epidemics of nonbacterial gastroenteritis. The noroviruses are common causes of waterborne epidemics of gastroenteritis, and have been shown to be responsible for outbreaks in nursing homes, on cruise ships, at summer camps, and in schools. Symptomatic treatment by itself is usually appropriate.

23. A 21-year-old primigravida at 10 weeks gestation has a negative titer for rubella. The best procedure to follow is to A) institute a -globulin regimen and maintain it throughout her pregnancy B) administer rubella vaccine after 12 weeks gestation C) administer rubella vaccine immediately post partum D) administer rubella vaccine 12 weeks post partum

ANSWER: C Rubella has been directly responsible for inestimable pregnancy wastage, as well as for severe congenital malformations. Identification and vaccination of unimmunized women immediately after childbirth or abortion is recommended. The use of (-globulin to prevent viremia in nonimmune subjects exposed to rubella is not recommended. The vaccine should be avoided shortly before or during pregnancy since it is an attenuated live virus. Because of herd immunity there is a very low likelihood that this patient will be exposed to rubella.

26. An 80-year-old female is admitted to your service at a skilled nursing facility 5 days after repair of a hip fracture. When you review her records you note that she has not received any previous treatment for osteoporosis. You are considering ordering zoledronic acid (Reclast) to reduce her risk of another fracture. Which one of the following should be evaluated before administering zoledronic acid to this patient? A) Vitamin D levels B) Liver enzyme levels C) Estimated glomerular filtration rate D) A CBC

ANSWER: C Secondary prevention of fractures is an important component of care following a hip fracture. Options to consider include bisphosphonates, calcium supplementation, and vitamin D supplementation. Bisphosphonates, including zoledronic acid, can reduce rates of clinical fractures among patients who have had a hip fracture (SOR A). While long-term use of bisphosphonates may increase the risk of jaw osteonecrosis and anemia, a CBC is not necessary before initiating therapy. Contraindications to zoledronic acid include hypocalcemia and a creatinine clearance <35 mL/min or other evidence of acute renal impairment.

37. Which one of the following cardiac rhythm abnormalities is most common in patients with anorexia nervosa? A) Atrial fibrillation B) Ventricular fibrillation C) Sinus bradycardia D) Sinus tachycardia E) Paroxysmal supraventricular tachycardia

ANSWER: C Sinus bradycardia is almost universally present in patients with anorexia nervosa. It is hypothesized that this is due to vagal hyperactivity resulting from an attempt to decrease the amount of cardiac work by reducing cardiac output. It is also possible that the bradycardia can be accounted for by low serum T-3 levels, a common finding in persons with chronic malnutrition. Sinus tachycardia may occur with refeeding in patients with anorexia. Other arrhythmias may also occur but are less frequent.

60. A mother meets you in the emergency department with her 3-week-old infant. The infant was delivered at term, with an uneventful prenatal and postnatal course to this point. The mother reports that the infant stopped breathing for 20-25 seconds, and that his lips and tongue appeared bluish. There was no coughing, choking, or congestion, but the child seemed "limp." The episode ended when the mother vigorously stimulated her child and he started crying. On examination, the child appears normal. Which one of the following would be most appropriate at this point? A) Reassurance and no further evaluation B) Discharge with a home apnea monitor C) Hospital admission for observation D) Mandatory referral to child protective services E) Direct laryngoscopy to rule out a foreign body

ANSWER: C Some experts recommend inpatient observation for all children with apparent life-threatening events such as this. Hospital admission is not always necessary, however, for a short, self-correcting episode associated with feeding. Given the history of not breathing for 20-25 seconds, having a blue tongue and lips, and being limp, admitting the child for observation is appropriate. Although child abuse is a concern, referral to child protective services is not mandatory. Laryngoscopy would not be routine, but might be appropriate in some cases depending on the history and physical findings. Many groups recommend home apnea monitoring after discharge for patients with more severe or undiagnosed cases.

99. A 25-year-old white male truck driver presents with a 1-day history of throbbing rectal pain. Your examination shows a large thrombosed external hemorrhoid. Which one of the following is the preferred initial treatment for this patient? A) Infrared coagulation B) Rubber band ligation of the hemorrhoid C) Elliptical excision of the thrombosed hemorrhoid D) Stool softeners and a topical analgesic/hydrocortisone cream

ANSWER: C The appropriate management of a thrombosed hemorrhoid presenting within 72 hours of the onset of symptoms is elliptical excision of the hemorrhoid and overlying skin under local anesthesia, such as 0.5% bupivacaine hydrochloride in 1:200,000 epinephrine, infiltrated slowly with a 27-gauge needle. Incision and clot removal may provide inadequate drainage, resulting in rehemorrhage and clot reaccumulation. Most thrombosed hemorrhoids contain multilocular clots that may not be accessible through a simple incision. Rubber band ligation is an excellent technique for management of internal hemorrhoids, and infrared coagulation is also used for this purpose. Banding an external hemorrhoid would cause exquisite pain. If the pain is already subsiding or more time has elapsed, and if there is no necrosis or ulceration, measures such as sitz baths, bulk laxatives, stool softeners, and local analgesia may be helpful. Some local anesthetics carry the risk of sensitization. Counseling to avoid precipitating factors such as prolonged standing/sitting, constipation, and delay of defecation is also appropriate.

74. An 82-year-old white male has a cardiopulmonary arrest while mowing his lawn and his heart rhythm is restored after 8 minutes of CPR by a neighbor. He is now your patient in the coronary care unit. He is on a ventilator and has severe hypoxic encephalopathy. Echocardiography shows an ejection fraction of 12% as a result of the massive anterior myocardial infarction he sustained. Your neurology consultant confirms that the patient will never again be able to meaningfully communicate, and will be ventilator-dependent. Prior to this, the patient had been living independently and had no health problems. He has no living relatives, and his attorney confirms that he has no written advance directives. The neighbor, who is a close friend, tells you that on several occasions recently he and the patient had discussed such a scenario, and that the patient had said that if he had little chance of a meaningful recovery he would not want to remain on life support. In consultation with the hospital ethics committee, which one of the following would be most appropriate in this case? A) Transfer care of the patient to another physician B) Ask a court to appoint a guardian to make medical decisions C) Withdraw life support D) Defer the decision regarding life support to the hospital attorney E) Ask the patient's attorney to decide whether to terminate life support

ANSWER: C The most common form of advance directive is a patient's conversations with relatives and friends, and these carry the same ethical and legal weight as written directives. Neither the hospital attorney nor the patient's personal attorney, in the absence of a previous discussion with the patient, has the ethical or legal authority to make the decision. Since the patient has previously expressed his wishes, it is unnecessary to have the court appoint a surrogate decision maker. Care should be transferred to another physician only if the original physician has a philosophical or religious objection to carrying out the patient's wishes.

38. An asymptomatic 32-year-old male requests screening for sexually transmitted diseases. A nucleic acid amplification test is performed on a urine sample, and the results are positive for gonorrhea and negative for Chlamydia. The patient has no known drug allergies. Which one of the following is the recommended treatment for this patient? A) Ceftriaxone (Rocephin), 125 mg intramuscularly B) Ceftriaxone, 250 mg intramuscularly C) Ceftriaxone, 250 mg intramuscularly, plus azithromycin (Zithromax), 1 g orally D) Ceftriaxone, 125 mg intramuscularly, plus doxycycline, 100 mg orally twice daily for 7 days E) Ciprofloxacin (Cipro), 500 mg orally

ANSWER: C The recommended treatment regimen for gonorrhea is ceftriaxone, 250 mg intramuscularly. The 125-mg regimen is no longer recommended because of treatment failures and limited effectiveness in pharyngeal infections. In addition, the patient should be given azithromycin, 1 g orally, because of the high incidence of coinfection with Chlamydia, even if testing is negative, and to decrease the risk for cephalosporin resistance.

44. A 56-year-old male with diabetes mellitus, hypertension, and chronic renal insufficiency presents for follow-up of his chronic medical conditions. Results of his most recent metabolic panel included an estimated glomerular filtration rate of 30 mL/min/1.73 m2 (N >60) and a calcium level of 10.4 mg/dL (N 8.5-10.2). Medication reconciliation reveals he is not taking the sevelamer (Renagel, Renvela) prescribed by the consulting nephrologist. You explain to the patient that he should be taking sevelamer to lower his serum calcium. The drug accomplishes this by A) blocking the effect of parathyroid hormone B) blocking excessive vitamin D levels, thus decreasing intestinal calcium absorption and increasing renal calcium excretion C) blocking intestinal absorption of phosphate, which lowers parathyroid hormone secretion D) directly blocking excessive calcium absorption in the intestines E) directly increasing the renal excretion of both calcium and phosphate

ANSWER: C This patient has secondary hyperparathyroidism, a common cause of hypercalcemia in patients with chronic renal insufficiency. Sevelamer is a newer synthetic agent in the therapeutic class of phosphate binders, which includes calcium acetate. Decreasing serum phosphate lowers the feedback stimulation of parathyroid hormone secretion by the parathyroid gland, which is often excessive in chronic renal insufficiency. Normalizing parathyroid levels improves serum calcium levels.

96. A morbidly obese 68-year-old male complains of breast enlargement. He has not noticed any pain or discomfort from this problem. His past medical history is negative except for type 2 diabetes mellitus and hypertension. His medications include metformin (Glucophage), 1000 mg twice daily; lisinopril (Prinivil, Zestril), 20 mg daily; and aspirin, 81 mg daily. His family history is negative for breast cancer. A physical examination is negative except for a BMI of 45 kg/m2 and symmetric bilateral adipose tissue in the breast region on inspection and palpation. There is no glandular tissue on careful palpation of the area beneath the areolae and nipples. No nodules or axillary nodes are detected. There is no nipple retraction or discharge, and no skin changes. Which one of the following is the most likely cause of this problem? A) Fat necrosis B) Gynecomastia C) Pseudogynecomastia D) Breast cancer E) Mastitis

ANSWER: C This patient most likely has pseudogynecomastia due to increases in subareolar fat secondary to his obesity. This is based upon clinical findings of symmetric adipose tissue in the breast region bilaterally and a lack of firm, palpable glandular tissue in the nipple and areolar region. In gynecomastia, there is palpable, firm glandular tissue in a concentric mass around the nipple-areola complex. Hard, immobile masses, masses associated with skin changes, nipple retraction, nipple discharge, or enlarged lymph nodes would suggest possible malignancy. Fat necrosis would involve a history of breast region trauma and would generally be asymmetric. Mastitis would cause clinical signs of infection.

32. Which one of the following is the usual reservoir for hantavirus? A) Prairie dogs B) Jackrabbits C) Deer mice D) Ground squirrels

ANSWER: C Up through July of 2013, the Centers for Disease Control and Prevention had logged 624 cases of hantavirus pulmonary syndrome in residents of 34 states. The infection killed more than a third of the victims. The virus is usually spread by inhalation of dried aerosolized deer mouse urine or droppings. Infected deer mice usually have few outward signs. Other hosts include the white-footed mouse, the rice rat, and the cotton rat. Other rodents have not been shown to carry the virus.

17. You receive a telephone call from the mother of a 5-year-old female. The child has had diarrhea and a decreased appetite for the past 2 days. She is still playing some. The mother reports no vomiting, but says her daughter has complained of a dry mouth and does not have tears when she cries. You suspect that the child may be mildly dehydrated. Which one of the following would you advise? A) Increased water intake B) Clear liquids with sodium, such as chicken broth C) An over-the-counter oral rehydration solution D) Intravenous fluids in the emergency department E) Loperamide (Imodium)

ANSWER: C When children show signs of dehydration from diarrhea, the first step is to assess its extent. In one study, four factors predicted dehydration: a capillary refill time >2 seconds, the absence of tears, dry mucous membranes, and an ill general appearance; the presence of two or more of these signs indicates a fluid deficit of at least 5%. This child has two of the signs, but does not require intravenous fluids at this point. Early oral rehydration therapy is recommended and can be started at home. This should be done using an oral rehydration solution that is designed for children (SOR C). Adult oral rehydration solutions should not be used in children. Water and other clear liquids, even those with sodium, such as chicken broth, should not replace an oral rehydration solution because they are hyperosmolar. These fluids do not adequately replace potassium, bicarbonate, or sodium, and can sometimes cause hyponatremia. Antidiarrheal medications are usually not recommended for use in children with acute gastroenteritis because they delay the elimination of infectious agents from the intestines.

67. A 62-year-old male underwent percutaneous coronary intervention and placement of two stents for a myocardial infarction yesterday. He is currently taking simvastatin (Zocor), aspirin, lisinopril (Prinivil, Zestril), and hydrochlorothiazide. His last LDL-cholesterol level was 70 mg/dL and his blood pressure is 130/80 mm Hg. Which one of the following additions to his current regimen would be most appropriate at this time? B) Amlodipine (Norvasc) C) Diltiazem (Cardizem) D) Verapamil (Calan, Verelan) E) Metoprolol (Lopressor, Toprol-XL) F) No changes

ANSWER: D -Blockers are first-line antihypertensive medications for patients with coronary artery disease (CAD) and have been shown to reduce the risk of death by 23% at 2 years. They should also be given to normotensive patients with CAD if tolerated. Cardioselective (1) -blockers such as metoprolol and atenolol are preferred, as they cause fewer adverse effects.

49. A 75-year-old male comes to your office for a routine follow-up visit for hypertension. He is asymptomatic but your evaluation reveals atrial fibrillation. An echocardiogram is normal except for mild left ventricular hypertrophy. Which one of the following would be most appropriate at this point? A) No anticoagulation B) Medical cardioversion C) Anticoagulation with aspirin D) Anticoagulation with warfarin

ANSWER: D This patient's age and his history of hypertension give him a CHADS2 score of 2, which implies that the patient is at a higher risk of thromboembolism and should be anticoagulated with warfarin and not aspirin. The patient should be fully anticoagulated for a minimum of 3 weeks prior to any attempt at cardioversion.

45. A 16-year-old afebrile, otherwise healthy female presents with a 4-day history of right ear pain. She says she has spent a fair amount of time swimming recently. Traction on the pinna causes pain. The erythema and inflammation is limited to the ear canal but there is too much edema to easily visualize the tympanic membrane. Which one of the following would be the most appropriate treatment? A) Amoxicillin B) Amoxicillin/clavulanate (Augmentin) C) Amoxicillin/clavulanate plus acetic acid 2% topically D) Ciprofloxacin 0.3%/dexamethasone 0.1% (Ciprodex) topically

ANSWER: D Acute otitis externa is treated with topical antibiotics. Although no preparation has been shown to be most effective, a fluoroquinolone does not create a risk of ototoxicity if the tympanic membrane is not intact. Topical corticosteroids may hasten symptom reduction. Oral antibiotics are not appropriate unless the infection has spread beyond the ear canal, or if the patient has diabetes mellitus or is immunocompromised.

71. A 52-year-old male with hypertension complains of increased dyspnea for the past 6 months. He reports that he has increased fatigue and dyspnea with normal activities. There is no cough or chest pain. He has a 30-pack-year history of smoking. On examination his blood pressure is 130/85 mm Hg, pulse rate 90 beats/min, respiratory rate 18/min, and O2 saturation 95% on room air. Heart sounds are normal with no murmurs. Auscultation of the lungs reveals bilateral rhonchi. In addition to ordering a chest radiograph, which one of the following should be performed next in the evaluation of this patient's dyspnea? A) A B-type natriuretic peptide (BNP) level B) A D-dimer level C) Arterial blood gas measurement D) Spirometry E) High-resolution CT of the chest

ANSWER: D Based on this patient's history and physical examination, COPD is the most likely cause of his dyspnea. Initial testing should include spirometry to diagnose airflow obstruction (SOR C). CT, a BNP level, a D-dimer level, and arterial blood gas measurements would not be the best initial tests in the evaluation of this patient's dyspnea.

46. An obese 10-year-old male with tonsillar hypertrophy is brought to your office because of snoring. There is no history of recent or past visits for tonsillitis. Polysomnography shows moderate obstructive sleep apnea syndrome. Which one of the following is the treatment of choice for this patient? A) Continuous positive airway pressure (CPAP) B) Intranasal corticosteroids C) Extended antibiotic therapy D) Adenotonsillectomy

ANSWER: D Childhood obstructive sleep apnea syndrome has a prevalence rate of 5.7%. It is associated with growth, cardiovascular, and neurobehavioral abnormalities. Adenotonsillectomy is the treatment of choice. Although CPAP can be effective, compliance is poor and it is therefore not a first-line treatment. Intranasal corticosteroids may also be helpful, but the benefit appears small.

28. A 68-year-old female presents with recent poor oral intake, fatigue, and confusion. Osmotic demyelination syndrome (central pontine myelinolysis) and permanent neurologic deficits could result from overly rapid correction of which one of the following abnormalities? A) Hyperglycemia B) Hyperkalemia C) Hypokalemia D) Hypernatremia E) Hyponatremia

ANSWER: E Overly rapid correction of hyponatremia may cause osmotic demyelination syndrome, or central pontine myelinolysis, sometimes resulting in permanent neurologic deficits after a brief improvement in neurologic status. Signs and symptoms may include dysarthria, dysphagia, paresis, coma, and seizures. It is believed that brain volume shrinks because it cannot assimilate the new electrolytes fast enough and water is lost from the cells. Rapid correction of hypernatremia that has been present for a short time is relatively safe. Hyperkalemia is a life-threatening condition that should be corrected promptly. Rapid correction of hypoglycemia is not an issue. Overly rapid correction of hyperglycemia and subsequent cerebral edema is unusual and is primarily seen in children.

10. Which one of the following is considered to be the highest strength of recommendation by the Strength of Recommendation Taxonomy (SORT) used by family medicine journals? A) Expert opinion B) A consensus guideline C) A retrospective cohort study D) Multiple good quality randomized, controlled trials

ANSWER: D Family medicine journals, including American Family Physician, The Journal of Family Practice, and The Journal of the American Board of Family Medicine utilize the Strength of Recommendation Taxonomy (SORT) to label key recommendations in clinical review articles. These grades are assigned on the basis of the quality and consistency of available evidence. The Cochrane Collaboration is an extensive database of systematic reviews and clinical trials. A Cochrane review with a clear recommendation warrants a strength of recommendation rating of A. This indicates consistent, good quality, patient-oriented evidence. Consistent findings from at least two randomized, controlled studies or a systematic review/meta-analysis of randomized, controlled trials are also assigned a level A strength of recommendation. Expert opinion and consensus guidelines are assigned a level C strength of recommendation. SORT also includes a grade of 1 to 3 for levels of evidence. Retrospective cohort studies are considered level 2.

30. A 40-year-old white male was seen 4 weeks ago for a sudden onset of cough and shortness of breath. At that visit his O2 saturation was 92%, but his examination and a chest radiograph were normal. You prescribed azithromycin (Zithromax) and an albuterol inhaler (Proventil, Ventolin). Ten days later he was feeling well and his oxygen saturation was 97%. Today he returns to the office with a dry cough and shortness of breath. On examination he has rare inspiratory rales that clear with deep breaths, and he has an O2 saturation of 86%. A chest film and a D-dimer test are normal. Pulmonary function tests show significant restriction that improves only minimally with albuterol. He has not been exposed to anyone with a similar illness, has no history of asthma, and has no smoking history or occupational exposure. However, he reports that 2 months ago his home was flooded after a heavy rain, and he has been tearing out carpeting that was ruined by the flood. Which one of the following is the most likely diagnosis? A) Persistent asthma with acute exacerbations B) Legionnaires' disease C) Pulmonary embolism D) Hypersensitivity pneumonitis

ANSWER: D Hypersensitivity pneumonitis can present in acute, subacute, or chronic forms. The case described includes two episodes of the acute form. The patient was exposed to mold antigens in his flooded home. Within 4-8 hours of exposure, chills, cough, and shortness of breath will be noted, and at times will be dramatic. A chest film can be normal, even with significant hypoxia. Pulmonary function tests will show restrictive changes, as compared to the reversible obstructive changes of acute asthma. Blood tests often show an elevated erythrocyte sedimentation rate. Serum IgG tests for the probable antigen confirm the diagnosis. Symptoms of acute hypersensitivity pneumonitis resolve over several days, but will suddenly and violently recur with repeated exposure to the offending antigen. The subacute form begins gradually over weeks or months, causing a cough and increasing shortness of breath. The chronic form develops over years of exposure, causing fibrotic changes to the lungs that will be evident on radiographs, as well as chronic crackles on auscultation. Asthma would be an unlikely diagnosis in this case, with the pulmonary function tests showing restrictive changes rather than obstructive changes, and little improvement with albuterol. Also, the lack of a previous history of asthma makes it less likely. Legionnaires' disease is always possible, but is unlikely in this case given the sudden onset, quick recovery over several days, and sudden recurrence. Pulmonary embolism is ruled out by the negative D-dimer test.

8. Which one of the following medications is most appropriate for treating moderate to severe shortness of breath in a hospice patient with lung cancer? A) Dexamethasone B) Haloperidol C) Scopolamine D) Morphine

ANSWER: D Morphine effectively decreases the feeling of shortness of breath in hospice patients. Randomized, controlled trials have shown significant improvements in symptoms without a significant change in oxygen saturation. Haloperidol can be used for nausea and vomiting (SOR B) and delirium, but is not helpful in the treatment of shortness of breath. Scopolamine is used to decrease the production of secretions but is not helpful for treating dyspnea. Corticosteroids will not manage the sensation of shortness of breath in a dying patient.

58. A 35-year-old male presents with acute low back pain after he spent a weekend building a storage shed in his backyard. He has no neurologic symptoms, and the pain does not radiate into either leg. Which one of the following has been shown to be a useful treatment in this situation? A) Bed rest B) Acupuncture C) Lumbar traction D) Cyclobenzaprine (Flexeril) E) Methylprednisolone (Medrol)

ANSWER: D Nonbenzodiazepine muscle relaxants such as cyclobenzaprine are beneficial for the relief of acute low back pain for the first 7-14 days after the onset of symptoms. Patient education, physical therapy, and the application of ice or heat may also help. Unsupported treatment options for acute low back pain include oral corticosteroids, acupuncture, lumbar support, massage, chiropractic spinal manipulation, and traction. Bed rest for acute low back pain is inadvisable.

86. A 54-year-old male presents with progressively worsening pain just below his right knee. He describes the pain as deep and aching, and says it is always present throughout the day, even while he is at rest, and worsens at night. Weight bearing intensifies the pain, as does heat. The patient does not recall any injury or other reason for the leg to hurt. He has not had any fever. His family history is positive for osteoarthritis in both parents when they were older, and an uncle has had a knee replacement. A physical examination is negative except for some varus deformity of the right lower extremity just below the knee. There is no redness. Radiographs demonstrate mild to moderate bowing of the proximal tibia. His alkaline phosphatase level is elevated but his -glutamyl transaminase level is normal. The remainder of a comprehensive metabolic panel is also normal. A CBC is normal, including the WBC count and differential. You should suspect which one of the following conditions? A) Osteoarthritis B) Osteoporosis C) Osteomyelitis D) Paget's disease of bone E) Seronegative spondyloarthritis

ANSWER: D Patients with bone pain caused by Paget's disease usually describe the pain as continuous. Unlike osteoarthritis, the bone pain of Paget's disease usually increases with rest, when the limbs are warmed, and at night. A variety of deformities may occur, including bowing of the tibia, and alkaline phosphatase is elevated. The case presented is not typical for osteoarthritis or osteoporosis, and the patient does not have a fever, elevated WBC count, or other findings suggestive of osteomyelitis. Seronegative spondyloarthritis (or spondyloarthropathy) is a family of inflammatory rheumatic diseases that cause arthritis. The most common is ankylosing spondylitis, which affects mainly the spine. Others include axial spondyloarthritis, which affects mainly the spine and pelvic joints; peripheral spondyloarthritis, affecting mostly the arms and legs; reactive arthritis (formerly known as Reiter's syndrome); psoriatic arthritis; and enteropathic arthritis/spondylitis associated with inflammatory bowel diseases (ulcerative colitis and Crohn's disease). The main symptom in most patients with spondyloarthritis is low back pain, which is most common in axial spondyloarthritis. In a minority of patients, the major symptom is pain and swelling in the arms and legs. This type is known as peripheral spondyloarthritis.

36. In women with polycystic ovary syndrome, the risk is increased the most for carcinoma of the A) breast B) cervix C) colon D) endometrium E) ovary

ANSWER: D Several disorders that are common in women with polycystic ovary syndrome are associated with an increased risk for endometrial carcinoma, including obesity, hyperinsulinemia, diabetes mellitus, anovulatory cycles, and high androgen levels.

87. A 62-year-old male with a 20-year history of diabetes mellitus presents with bilateral calf and buttock pain that occurs after he walks 2 blocks. The symptoms are relieved with rest. On examination his pedal pulses are not palpable and his ankle-brachial index is 1.45. Which one of the following would be most appropriate? A) Reassuring the patient that his ankle-brachial index is normal B) MRI of the lumbar spine C) A repeat evaluation in 6 months if the symptoms persist D) MR or CT angiography of the lower extremities

ANSWER: D The National Health and Nutrition Examination Survey (NHANES) found that 1.4% of adults over 40 have an ankle-brachial index (ABI) >1.4; this group accounts for approximately 20% of all adults with peripheral artery disease. An ABI >1.4 indicates noncompressible arteries (calcified vessels). In patients with arterial calcification, such as diabetic patients, more reliable information is often obtained by using toe pressures to calculate a toe-brachial index and from pulse volume recordings. Vascular imaging should be used to confirm peripheral vascular disease. MR or CT arteriography, duplex scanning, and hemodynamic localization are noninvasive methods for lesion localization and may be helpful when symptoms or findings do not correlate with the ABI. Contrast arteriography is used for definitive localization before intervention.

65. In which one of the following populations does the U.S. Preventive Services Task Force support ultrasound screening for abdominal aortic aneurysm? A) All men age 55-75 B) Males age 55-75 who currently smoke C) Patients of both sexes age 55-75 who currently smoke D) Men age 65-75 who have ever smoked E) No population group

ANSWER: D The U.S. Preventive Services Task Force (USPSTF) recommends one-time screening for abdominal aortic aneurysm (AAA) by ultrasonography in men age 65-75 who have ever smoked (Grade B recommendation). The USPSTF recommends against routine screening for AAA in women (Grade D recommendation).

91. A 70-year-old alcoholic male is recovering from a myocardial infarction. On the fourth hospital day he complains of a sudden onset of excruciating abdominal pain that is not significantly reduced by large doses of morphine. He becomes nauseated, begins to vomit, and has diarrhea. The patient appears agitated and confused, and his heart rate increases. He also becomes hypotensive. Physical examination of his abdomen reveals minimal tenderness, decreased bowel sounds, and a moderately enlarged liver. Laboratory Findings WBCs 17,600/mm3 with a left shift (N 4300-10,800) BUN 40 mg/dL (N 8-25) Creatinine 1.0 mg/dL (N 0.6-1.5) Serum lipase 150 U/L (N 0-160) Arterial pH 7.14 (N 7.35-7.45) The most likely diagnosis is A) alcohol withdrawal syndrome B) pulmonary embolus C) pancreatitis D) acute mesenteric artery embolism E) perforated gastric ulcer

ANSWER: D The hallmark of acute mesenteric artery ischemia is severe abdominal pain that is out of proportion to physical findings. This is a life-threatening event, which often follows a myocardial infarction when a mural thrombus occludes a superior mesenteric artery. Patients rapidly become acidotic and hypotensive, and experience a high mortality rate. Alcoholic withdrawal syndrome has a much more insidious onset, with tremors, agitation, and anxiety being the prominent features. Abdominal pain is not a common prominent symptom or finding. Although pulmonary embolism is possible in the patient described, the major symptoms are dyspnea and chest pain, and abdominal pain and tenderness would not be a typical finding. A normal serum lipase level should point to a nonpancreatic origin for this patient's problem. A perforated gastric ulcer would typically have a more insidious onset and the abdominal examination would usually demonstrate marked focal tenderness.

55. A 45-year-old female with no significant past medical history presents to your office with 2 weeks of worsening pain in her right arm. For the past 2 months she has worked on a plastics manufacturing assembly line. A physical examination reveals no swelling and a normal range of motion. She has normal strength in the upper extremity but she experiences increased pain with extension of her right wrist against resistance. Palpation reveals marked tenderness over the lateral epicondyle of the right arm. Which one of the following is most likely to improve the patient's long-term outcome? A) Physical therapy B) Regular physical activity using her hands and arms C) Use of an inelastic, nonarticular proximal forearm strap D) Modifying her work routines

ANSWER: D This patient has signs and symptoms of lateral epicondylitis, also known as tennis elbow, or alternatively as lateral epicondylalgia to reflect the noninflammatory nature of the condition. This is an overuse tendinopathy of the common extensor tendon origin of the lateral elbow. Conservative care that includes offloading the involved tendons is the key to improving outcomes at 1 year, which would mean modifying this patient's work. Physical therapy can improve pain and function in the short term, but has not been shown to improve long-term outcomes at 1 year in randomized trials (SOR A). The evidence is weaker for bracing, with some studies showing improved pain and function at 3-6 weeks (SOR B). Recent randomized, controlled trials have made it clear that while corticosteroid injections reduce acute pain for up to 6 weeks, their use increases rates of poor long-term outcomes (SOR A).

3. You have just finished giving a prescription with instructions to a 28-year-old male from El Salvador who speaks limited English. You gave the instructions with the aid of an interpreter, but are concerned that the patient might not fully understand them. Which one of the following is the best course of action? A) Refer the patient to a website about his condition B) Repeat the instructions slowly to the interpreter and ask him or her to speak clearly to the patient C) Contact a family member who speaks English and ask him or her to repeat the instructions to the patient D) Ask the patient to repeat the instructions to you in his own words

ANSWER: D To ensure that patients from other cultures understand instructions, it is helpful to ask them to repeat the instructions in their own words. A website would probably not be specific or culturally sensitive to the patient's condition. The physician should speak in a normal tone to the patient, and not to the interpreter. Family members may be used as convenient translators if necessary, but to maintain confidentiality and reduce miscommunication it is best to use a trained medical interpreter.

47. For which type of renal calculus is acidification of the urine indicated? A) Cystine B) Uric acid C) Calcium oxalate D) Calcium phosphate

ANSWER: D Urine pH is an important factor in the production of kidney stones. Uric acid, cystine, and calcium oxalate stones tend to form in acidic urine, whereas struvite (magnesium ammonium phosphate) and calcium phosphate stones form in alkaline urine. Urine should be acidified for prevention of calcium phosphate and struvite stones. Cranberry juice or betaine can lower urine pH.

14. Activated protein C resistance (factor V Leiden) is most commonly found in patients with A) hemolytic anemia B) carcinoma of the lung C) familial hypercholesterolemia D) venous thrombotic disease E) cystic fibrosis

ANSWER: D Venous thrombosis, both acute and recurrent, is associated with several hematologic abnormalities, in addition to the well-known factors of trauma, surgery, malignancy, sepsis, and oral contraceptive use. Notably, activated protein C resistance (factor V Leiden) has been found to be one of the most common hereditary causes of thrombophilia.

93. A 55-year-old male has New York Heart Association class III chronic systolic heart failure due to hypertensive cardiomyopathy. Which one of the following is CONTRAINDICATED in this patient? A) Carvedilol (Coreg) B) Digoxin C) Ramipril (Altace) D) Spironolactone (Aldactone) E) Verapamil (Calan)

ANSWER: E ACE inhibitors and Beta-blockers improve mortality in heart failure (HF). Digoxin and furosemide improve symptoms and reduce hospitalizations in systolic HF, and furosemide may decrease mortality. Spironolactone, an aldosterone antagonist, reduces all-cause mortality and improves ejection fractions in systolic HF. Verapamil, due to its negative inotropic effect, is associated with worsening heart failure and an increased risk of adverse cardiovascular events.

97. Which one of the following is most likely to cause hypoglycemia in elderly patients? A) Metformin (Glucophage) B) Pioglitazone (Actos) C) Glipizide (Glucotrol) D) Sitagliptin (Januvia) E) Glyburide (DiaBeta)

ANSWER: E Among the oral antiglycemic drugs, the sulfonylurea agents are the most likely to cause hypoglycemia, and glyburide is more likely to cause hypoglycemia than glipizide. Glyburide should rarely be used in the elderly.

24. A postmenopausal female who has recently been diagnosed with hypertension returns for follow-up 3 months after the initiation of therapeutic lifestyle changes. Her blood pressure has improved but remains higher than goal at 142/90 mm Hg, and pharmacologic treatment is indicated. The patient has a family history of osteoporosis. Which one of the following may slow the demineralization of bone in this patient? A) An ACE inhibitor B) An -blocker C) A -blocker D) A calcium channel blocker E) A thiazide diuretic

ANSWER: E An adult patient with a confirmed systolic blood pressure >139 mm Hg or a diastolic blood pressure >89 mm Hg is hypertensive. JNC-7 guidelines recommend the adoption of healthy lifestyles for all patients, especially those with hypertension, and the addition of pharmacologic treatment as necessary to reach a goal blood pressure <140/90 mm Hg. This goal blood pressure is further reduced to <130/80 mm Hg for patients who also have diabetes mellitus or renal disease. The same guidelines note that elevated systolic blood pressure is a much more important cardiovascular disease risk factor than diastolic blood pressure in persons older than age 50. Medication is commonly required to reach the goal blood pressure, and most patients will often require two or more drugs. In the absence of compelling indications for use of a specific class of drugs, thiazide-type diuretics are recommended for initial treatment. Examples of compelling indications include ACE inhibitors for patients with heart failure, diabetes mellitus, or high coronary disease risk, or Beta-blockers post myocardial infarction. The selection of an agent with favorable side benefits is recommended. Thiazide-type diuretics are useful in slowing demineralization from osteoporosis, making this the most appropriate choice for this patient. Other examples of choosing drugs based on side benefits include Beta-blockers for patients with a history of migraine or tachycardia, calcium channel blockers for patients with Raynaud's syndrome, and "-blockers for patients with benign prostatic hyperplasia.

61. A 30-year-old female at 36 weeks gestation has a positive culture for group B Streptococcus. Her past medical history is significant for the development of a nonurticarial rash in response to penicillin. Which one of the following is most appropriate for intrapartum antibiotic prophylaxis in this patient? A) Azithromycin (Zithromax) B) Clindamycin (Cleocin) C) Vancomycin (Vancocin) D) Ampicillin E) Cefazolin

ANSWER: E Cefazolin is appropriate for intrapartum prophylaxis against group B Streptococcus (GBS) in penicillin-allergic patients who do not have a history of anaphylaxis, urticaria, angioedema, or respiratory distress. Depending on the antibiotic sensitivity of the GBS organism, either vancomycin or clindamycin is recommended for patients at higher risk for anaphylaxis.

13. A 47-year-old postmenopausal female falls while carrying groceries into her house and sustains a right distal radial fracture. A chemistry panel reveals a calcium level of 11.2 mg/dL (N 8.6-10.6) and further evaluation leads to a diagnosis of primary hyperparathyroidism. Which one of the following is the best course of treatment for this patient? A) Estrogen replacement therapy B) Long-term bisphosphonate therapy C) Daily furosemide treatment with increased oral fluids D) Elimination of calcium and vitamin D from the diet E) Referral to a surgeon for consideration of parathyroidectomy

ANSWER: E Hyperparathyroidism is usually caused by a single adenoma of one of the four parathyroid glands. A minority of cases (10%-15%) are associated with four-gland hyperplasia. Studies that localize the glands, such as a technetium scan or ultrasonography, help surgeons who are familiar with this condition achieve a cure rate of 95%-98%, with an estimated complication rate of 1%-3%. For patients <50 years old or symptomatic patients, such as those with a fragility fracture, parathyroidectomy is the treatment of choice. If a patient is older, is a poor surgical candidate, or has asymptomatic disease, long-term monitoring with treatment focused on reducing bony complications can be considered (SOR C).

20. An abandoned infant is brought to the hospital for evaluation. Based on the presence of a dried umbilical cord remnant and her overall appearance, you believe her to be no more than 5 days of age. A thorough examination is normal except for a finding of bilateral conjunctival erythema and exudate. A Gram stain of the exudate is remarkable for numerous WBCs, very few of which are noted to contain gram-negative diplococci. Which one of the following treatment options is most appropriate? A) Application of moist, warm saline eye compresses B) Irrigation of both eyes with povidone-iodine (Betadine) C) One-time application of ophthalmic erythromycin ointment into both eyes D) Instillation of silver nitrate solution into both eyes E) Intramuscular injection of ceftriaxone (Rocephin)

ANSWER: E Infantile gonococcal infection is usually the result of exposure to infected cervical exudate during delivery and manifests 2-5 days after birth. Ophthalmia neonatorum and sepsis are the most severe gonococcal infections in newborns and immediate treatment is warranted based on the presumptive diagnosis. Topical antibiotics are appropriate for prophylaxis, but not for treatment. Silver was used for prophylaxis at one time, but is no longer available. Povidone-iodine has not been studied for prevention. A single dose of 25-50 mg/kg of ceftriaxone administered intravenously or intramuscularly is the recommended treatment.

54. A 39-year-old male with a BMI of 41 kg/m2 is interested in weight loss. His medical history includes adequately controlled type 2 diabetes mellitus, well-controlled hypertension, hyperlipidemia, and obstructive sleep apnea. He has no history of coronary artery disease or COPD. Which one of the following is likely to be most effective for long-term weight loss in this patient? A) A very low calorie diet B) Increased physical activity C) Frequent, long-term weight-loss counseling D) Pharmacotherapy E) Bariatric surgery

ANSWER: E Obesity increases the risk of a variety of medical conditions, including type 2 diabetes mellitus, hypertension, hyperlipidemia, pulmonary disease, coronary artery disease, gallstones, fatty liver disease, obstructive sleep apnea, GERD, osteoarthritis, and a variety of forms of cancer. A weight loss of at least 10% for greater than 1 year leads to statistically significant improvement in lipid ratios, blood glucose homeostasis, and coronary artery disease risk reduction. The AAFP recommends screening for obesity and intensive counseling (more than 1 session per month for more than 3 months) with behavior modification for obese patients. Counseling is ineffective by itself and must be combined with lifestyle modification. Dietary modification, increased physical activity, and behavior modification are effective for maintaining modest weight loss for greater than 1 year (SOR B). However, there are few large, randomized, controlled trials with subjects maintaining weight reductions of 10% for over 1 year, even when combining therapy, exercise, and dietary restriction. Long-term pharmacotherapy can lead to weight loss, but regaining some weight is typical. Bariatric surgery leads to the most effective weight reduction and long-term maintenance in patients who are morbidly obese (SOR A). Gastric bypass is effective, with a mean weight loss of 71.2% at 3 years; with laparoscopic gastric banding the mean weight loss is 55.2% at 3 years. In one study, 94% of gastric bypass patients maintained at least a 20% weight loss at 6 years. Bariatric surgery has also been shown to significantly reduce fasting blood glucose, with resolution of diabetes mellitus in 31%-77% of lap band patients and 72%-100% of gastric bypass patients. Bariatric surgery is a safe and effective means for long-term weight loss and should be considered in adults with a BMI >40, or >35 with obesity-related comorbidities.

63. Pretibial myxedema is a cutaneous manifestation of A) subclinical diabetes mellitus B) collagen vascular disease C) hyperlipidemia, type III D) ischemia E) Graves disease

ANSWER: E Pretibial myxedema is a complication of Graves disease, whether it presents as hypo- or hyperthyroidism. It is a dermopathy that most often occurs in the lower legs and results from increased deposition of mucin due to the endocrine abnormality. Diabetes mellitus can cause necrobiosis lipoidica, a lesion on the lower extremities; hyperlipidemia can cause waxy papules; and collagen vascular and ischemic disease can cause urticaria and/or ulceration.

52. A 13-year-old African-American male is brought to your office by his mother for a limp that she has noticed for about 1 week. The patient admits to vague right-sided hip and knee pain present only with activity. He says the pain has never awakened him from sleep and is never present at rest. Neither the mother nor the patient has noticed any systemic symptoms such as fever, night sweats, weight loss, or appetite changes. There is no recent history of trauma. On examination the patient's weight is in the 90th percentile and his height is in the 50th percentile. He has an antalgic gait. Examination of the left hip is normal. Examination of the right hip is significant for decreased internal rotation. The right hip externally rotates involuntarily with passive flexion. There is no external deformity and no skin changes are noted. He has a negative FABER test. Palpation of the bursa and bone does not elicit pain. Examination of both knees is normal. Plain radiographs confirm your clinical impression. Which one of the following is the most appropriate next step in the management of this patient? A) Reassurance with close follow-up B) Physical therapy C) Injection of the sacroiliac joint D) Hospital admission for intravenous antibiotics E) Surgery

ANSWER: E The most likely diagnosis for this patient is stable slipped capital femoral epiphysis (SCFE). This is more frequent in males than in females, and is more common in African-Americans and Pacific Islanders than in whites. Although some patients present with pain, many present with a painless limp or vague pain. The average age of onset is 13.5 years for males and 12 years for females. Obesity is strongly associated with SCFE. The lack of systemic symptoms makes osteomyelitis, abscess, or a septic joint much less likely. Malignancy is a possibility, but night pain would be more likely. Sacroiliitis is much less likely given a negative FABER test. The patient's age makes transient synovitis or Legg-Calvé-Perthes disease less likely. Although muscle strain is a possibility, the physical examination findings of external rotation deformity and limited internal rotation are more specific for SCFE. Once the diagnosis of SCFE is made, the patient should not bear weight and should be referred promptly for surgery to prevent complications.

34. A new home urine test is designed to detect a particular type of cancer. The gold standard test for this cancer is a biopsy, but a biopsy is more costly than the urine test, is invasive, and is associated with a number of adverse side effects. To test the effectiveness of the home urine test, 104 people took the test and then agreed to a biopsy. When the study was concluded, 77 people tested negative and 27 tested positive on the urine test. Biopsies were positive in 18 individuals, 8 of whom tested negative on the urine test. What is the negative predictive value of the home urine test, rounded to a whole number? A) 20% B) 37% C) 56% D) 80% E) 90%

ANSWER: E The results of this urine test were that 10 people had the disease and tested positive (true positives); 8 people had the disease but tested negative (false negatives); 17 people did not have the disease (27 - 10) but tested positive (false positive); 69 people did not have the disease (77 - 8) and tested negative (true negative). The negative predictive value is determined by dividing the true negatives (69) by the total number who tested negative (true plus false negatives = 77). The result is 89.6%, which rounded to a whole number is 90%.

73. A 21-year-old male presents with a complaint of headaches for the past 6 months. He has severe, sharp, right-sided periorbital pain 3-4 days each week. When these headaches occur his right eye gets watery, his right nostril feels clogged, and his forehead feels sweaty. When he gets the headaches he takes four 200-mg ibuprofen tablets and goes into a dark, quiet room. The headaches usually resolve in about 90 minutes. Currently he is feeling well and his examination is completely normal. What type of headache does he most likely have? A) Medication overuse headache B) Migraine C) Paroxysmal hemicrania D) Temporal arteritis E) Cluster headache

ANSWER: E This patient has cluster headaches. Most people with cluster headaches are male. These headaches typically present with severe unilateral pain that lasts from 15 minutes to 3 hours. The pain is generally extremely sharp, continuous, and incapacitating. In addition to the pain, the headaches are associated with at least one of the following ipsilateral signs: conjunctival injection, lacrimation, nasal congestion, miosis or ptosis, eye edema, and forehead and facial sweating. Patients may also have a sense of restlessness or agitation. The headaches occur anywhere from every other day up to 8 times a day, often in cycles for 4-12 weeks. Cluster headaches respond to most of the same medications as migraine headaches (DHE, ergotamines, triptans). They also respond well to 100% oxygen therapy. Paroxysmal hemicranias are very unusual and present with a similar type of pain, but the attacks are usually short and they are more common in women. Medication rebound headaches tend to be diffuse, bilateral, almost daily headaches. These occur in people who are overusing medications, and they tend to get worse with physical or mental exertion. Temporal arteritis usually occurs in older adults. Migraines are also often unilateral but they are usually pulsatile, and are associated with nausea and vomiting or photophobia and phonophobia.

98. A 42-year-old female is troubled by her lack of interest in sex. She is generally healthy, takes no medications, and has regular menstrual periods. She is content with the emotional intimacy of her marriage and has had satisfying sexual interactions in the past. She does not have any religious or cultural barriers regarding her sexuality, and asks for ideas on how to improve her situation. Which one of the following has consistent evidence of benefit in cases such as this? A) Cognitive-behavioral therapy B) Viewing pornography C) Oral estrogen D) Oral sildenafil (Viagra) E) Topical testosterone

ANSWER: E This patient meets the criteria for hypoactive sexual desire disorder (HSDD). The incidence of this condition is variable based on the age, life stage, and culture of the patient, but is estimated to be present in about 5%-15% of the adult female population. This diagnosis includes two components: (1) recurrent deficiency or absence of sexual desire or receptivity to sexual activity, and (2) distress about such a deficiency. In menstruating women, oral estrogen and oral sildenafil have not been shown to be superior to placebo. Cognitive-behavioral therapy has been shown to be helpful for other sexual dysfunctions, but not with HSDD. Topical testosterone, in either patch or gel form, has shown consistent improvements in arousal, desire, fantasy, orgasm, and overall satisfaction in cases of HSDD.

1. A 52-year-old female with a history of hypertension and hypercholesterolemia presents with mild edema, weakness, and body aches. Her only medications are atorvastatin (Lipitor) and chlorthalidone. Her previously normal serum creatinine level is now 2.6 mg/dL (N 0.64-1.27). Her BUN level is 32 mg/dL (N 6-20) and her serum is clear without pigmentation. The urine dipstick is positive for blood, but a microscopic examination is negative for WBCs, RBCs, and casts. The most likely diagnosis is A) allergic interstitial nephritis B) glomerulonephritis C) hemolysis D) pyelonephritis E) rhabdomyolysis

ANSWER: E This patient with acute kidney injury (AKI) has clinical symptoms and signs consistent with rhabdomyolysis, a known cause of AKI. Furthermore, she is taking a medication known to cause rhabdomyolysis. The urinalysis with a positive dipstick for blood and no RBCs on the microscopic examination is indicative of either hemolysis or rhabdomyolysis. Darkened, pigmented serum would be expected with hemolysis, while rhabdomyolysis is associated with clear serum. Urine abnormalities found in glomerulonephritis include proteinuria and RBC casts, while patients with allergic interstitial nephritis may have eosinophils and possibly WBC casts. Pyelonephritis is associated with WBCs in the urine, and if the dipstick is positive for blood there will be RBCs on the microscopic examination.

16. A 43-year-old female presents to your office 2 days after discovering a rash on her back, shown below. Which one of the following treatments will decrease her chances of developing long-term sequelae? A) Amitriptyline B) Gabapentin (Neurontin) C) Oral corticosteroids D) Topical corticosteroids E) Oral acyclovir (Zovirax)

ANSWER: E While some studies have shown mixed results, there is good evidence that oral acyclovir reduces the incidence of herpetic neuralgia when given within 72 hours of the onset of the rash, and that it reduces the duration of symptoms (SOR A). Acyclovir, valacyclovir, and famciclovir have also been shown to reduce the formation of new lesions, reduce viral shedding, and hasten the resolution of lesions. The effect of acyclovir on preventing neuralgia appears to be strongest in the first month. Oral or topical corticosteroids can reduce the duration of the rash and pain in the acute phase. Tricyclic antidepressants and gabapentin can be used to treat the pain of postherpetic neuralgia if it does develop (SOR A).


Set pelajaran terkait

Macroeconomics Exam 2 Launchpad HW

View Set

Combo with Mnemonics and 27 others_10-25

View Set

control of gene expression in prokaryotes ch16

View Set

Introduction to business unit one exam, (chapters 1-7)

View Set